1. Trang chủ
  2. » Sinh học lớp 12

Tạp chí Toán tuổi thơ kỳ số 204 và 205

68 53 0

Đang tải... (xem toàn văn)

Tài liệu hạn chế xem trước, để xem đầy đủ mời bạn chọn Tải xuống

THÔNG TIN TÀI LIỆU

Nội dung

Let a, b be the two numbers. CÇu GiÊy, Hµ Néi.. KiÓu tÝnh l·i nh− trªn gäi lµ l·i kÐp.. Th¸m tö Sª Lèc Cèc.. Cho h×nh vu«ng ABCD. Gäi O lµ giao ®iÓm cña AC vµ EG. Gäi K lµ giao ®iÓm cña [r]

(1)(2)(3)(4)

Đề thi Olympic lớp quận Hoàng Mai, TP Hà Nội 25 Dành cho nhà toán học nhỏ 29 Sử dụng tính chất hàm s bc nht gii toỏn

Thái Nhật Phợng

Kết Thi giải toán qua th− 32 Giải toán nào? 38 Chứng minh đ−ờng thẳng qua điểm cố định Nguyễn Xuân Cảnh

Sai đâu? Sửa cho 43 Lời giải đ−ợc cha?

Nguyễn Thanh Giang

Dạy - Học chơng trình giáo dục phổ thông - Chơng trình 2018

Những yêu cầu cần đạt lớp 44 Câu lạc Tốn Tuổi thơ 48 Kì 32

Thái Nhật Phợng

Vào thăm Vờn Anh 49 Các quốc gia Châu

Nguyễn Thị HiỊn

V−ợt vũ mơn 50 Tốn tổ hợp đề thi chọn học sinh giỏi tuyển sinh vo lp 10 chuyờn

Nguyễn Đức Tấn

Lịch sử Toán học 55 Sai phân pháp sách toán chữ Hán Đại thành toán học minh

(Tiếp theo TTT2 số 200+201)

Tạ Duy Phợng, Phạm Vũ Lộc, Đoàn Thị Lệ, Cung Thị Kim Thành, Phan Thị ánh Tuyết Đề thi nớc 58 Australian Mathematics Competition 2019 Đỗ Trung Kiên

(5)

Problem 11(204+205) How many 3-digit numbers are there that have odd number of factors?

TS Đỗ ĐứC THàNH Trờng liên cấp Tiểu học THCS Ngôi Sao Hà Nội Các bạn hÃy giải toán tiếng Anh gửi tòa soạn nhé! Năm bạn có giải tốt đợc nhận quà

Problem 9(202) Let a, b be the two numbers a + b = 100 Therefore, ab = a(100 − a) = 100a − a2 = 2500 − 2500 + 100a − a2 = 2500 − (a2 − 100a + 2500) = 2500 − (a − 50)2

As (a − 50)2 non-negative Consequently,ab ≤ 2500

Nhận xét Chúc mừng bạn sau có lời giải đúng: Trần Trung Phúc, 8A4, THCS Ngơ Gia Tự, Hồng Bàng, Hải Phịng; Bạch Thái Sơn, 7A1, THCS Vĩnh Yên, Vĩnh Yên, Vĩnh Phúc; Nguyễn Ngọc Diệu, 9A, THCS Nguyễn Hiền, Nam Trực, Nam Định; Phạm Thùy Trâm, 7C, THCS Bạch Liêu, Yên Thành, Nghệ An; Nguyễn Minh Hoàng, 7E, THCS Nguyễn Trãi, Nghi Xuân, Hà Tnh

Các bạn sau có lời giải tốt đợc khen: Nguyễn

Mạnh Hùng, 9A3, THCS Giấy Phòng Châu, Phù Ninh, Phú Thọ; Trơng Quang Mạnh, 9D, THCS Đặng Thanh Mai, TP Vinh, Nghệ An; Nguyễn Trần Kiên, 7C1, THCS Archimedes Academy, Q Cầu Giấy, Hà Nội

Đỗ §øc thµnh

Thơng báo thi Câu lạc Tốn Tuổi thơ tồn quốc 2020

Từ năm 2005 đến năm 2014, tạp chí phối hợp với Sở Giáo dục Đào tạo tổ chức đ−ợc lần Giao l−u Toán Tuổi thơ lần Olympic Tốn Tuổi thơ tồn quốc Từ năm học 2015 - 2016 đến nay, tạp chí Tốn Tuổi thơ phối hợp với Sở Giáo dục Đào tạo tổ chức thi Câu lạc Tốn Tuổi thơ tồn quốc thành công tỉnh (thành): Hà Nội (năm 2016), Trà Vinh (năm 2017), Lào Cai (năm 2018), thành phố Đà Nẵng (năm 2019) Đây sân chơi trí tuệ, lành mạnh, bổ ích cho em học sinh thầy cô giáo Sở Giáo dục Đào tạo Nghệ An có cơng văn số 17/SGD&ĐT - GDTH ngày 03/01/2020 việc nhận lời mời đăng cai thi Câu lạc Tốn Tuổi thơ tồn quốc năm 2020 Cuộc thi đ−ợc tổ chức vào tuần thứ hai tháng Mọi thông tin thi đ−ợc công bố trang điện tử http://www.toantuoitho.vn số tạp chí hàng tháng

(6)

Máy gì?

Máy lợn trời

Máy xẻ gỗ giúp ng−ời nhanh Máy đồng hạn mừng rơn Máy vải vóc đẹp trơn

Máy may áo đáng yêu

Máy nghịch ngợm ngời kêu rụt vào Máy nâng vật lên cao

Máy cắt lúa, rào rào thóc Máy làm mát phòng ta

Máy nguyên tắc khéo mà phiền Mong thần dân đoán thật mau Gửi bài, đợi Trẫm số sau thng lin!

Lê Việt Ngọc (Thanh Xuân, Hà Néi)

(TTT2 sè 202)

Loμi chim no?

Chim én báo hiệu xuân sang

B cõu biểu t−ợng hịa bình gian Chào mào đội mũ xinh Chim cơng hay múa lung linh xịe

Chích chòe tên có vần oe Tu hú mùa hè hay kêu

Bói cá chộp cá rÊt siªu

Chim vĐt cã tÝnh nghe nhiỊu nãi theo Cú mèo mặt giống nh mèo Chim chèo bẻo vần eo kì

Họa mi tiếng hót mê li

Cánh cụt bớc, chẳng phi lên trêi

Nhận xét Kì Vua Tếu ban th−ởng cho bốn thần dân làm đúng, trình bày đẹp có tên sau đây: Nguyễn Thanh Sơn, 6C, THCS Đặng Thai Mai, TP Vinh, Nghệ An; Nguyễn Th−ờng Phong, 7A, THCS Hàn Thuyên, L−ơng Tài, Bắc Ninh; Phạm Tuấn Minh, 7B, THCS Tân Bình, TP Tam Điệp, Ninh Bình; Bùi Phạm Ph−ơng Anh, 7A2, THCS Mộc Lỵ, Mộc Châu, Sơn La

(7)

TìM Số CòN THIÕU!

Cho b¶ng sè nh− sau:

Hãy chọn đáp án để điền vào dấu chấm hỏi (?) cho hợp lơgic

đỗ thị thúy ngọc Phịng Giáo dục Trung học, Sở GD&ĐT Ninh Bình (S−u tầm v gii thiu)

số thích hợp?

(TTT2 số 202)

Đáp án Có hình trịn nhỏ tạo thành cặp đối xứng qua tâm hình trịn lớn Mỗi số hình trịn nhỏ tổng hai số liền sát hình trịn đối xứng, ? = + = 17

Nhận xét Quy luật kì dễ nên tất bạn chọn kết Một số bạn diễn đạt ch−a rõ ràng Xin trao th−ởng cho bạn: D−ơng Nh− Quỳnh, 7A, THCS Lý Tự Trọng, Bình Xuyên, Vĩnh Phúc; Hoàng Minh Vũ, 7D, THCS Văn Lang, TP Việt Trì, Phú Thọ; L−u Thùy D−ơng, 8A3, THCS Trần Đăng Ninh, TP Nam Định, Nam Định; Trần Trung Phúc, 8A4, THCS Ngơ Gia Tự, Hồng Bàng, Hải Phịng; Lê Phú Quang, 7D, THCS Nhữ Bá Sỹ, Hoằng Hóa, Thanh Húa

Các bạn sau đợc tuyên dơng: An Hà Hùng, 7A1, THCS Mộc Ly, Mộc Châu, Sơn La; Ngun C«ng Vinh, Ngun Anh Th−, 7C, THCS Ngun Cao, Quế Võ, Bắc Ninh; Lê Thị Diệu Thúy, 9A, THCS Bình Thịnh, Đức Thọ; Đặng Đình Khánh, 8/2, THCS Lê Văn Thiêm, TP Hà Tĩnh, Hà Tĩnh; Ngô Bích Loan, 7C, THCS Bạch Liêu, Yên Thành, Nghệ An

(8)

Bài toán lÃi suất ngân hàng

NGUYễN TIếN HợP

Trờng THCS Tân Thợng, Văn Bàn, Lào Cai

Tr−íc hÕt, ta t×m hiĨu mét sè h×nh thøc tÝnh l·i suÊt nh− sau:

Lãi đơn: Lãi đ−ợc tính theo tỉ lệ phần trăm khoảng thời gian cố định tr−ớc Ví dụ: Khi ta gửi 50 triệu đồng vào ngân hàng với lãi suất 7% /năm sau năm số tiền lãi nhận đ−ợc

50 7% = 3,5 (triệu đồng)

Sau hai năm số tiền gốc lẫn lãi 50 + 3,5 = 57 (triệu đồng)

Sau n năm số tiền gốc lẫn lãi 50 + n 3,5 (triệu đồng)

Kiểu tính lãi nh− gọi lãi đơn

Lãi kép hay lãi gộp: Đ−ợc hiểu đơn giản tái đầu t− lãi, tức sau sinh lời, lãi đ−ợc dồn vào tiền vốn để tiếp tục cho chu kì đầu t− tất nhiên, vốn gửi nhiều lãi lại cao giai đoạn sau Sau đơn vị thời gian (kì hạn), tiền lãi đ−ợc gộp vào vốn tính lãi Câu “lãi mẹ đẻ lãi con” từ kiểu tính lãi

Ví dụ: Khi ta gửi 50 triệu đồng vào ngân hàng với lãi suất 7% /năm sau năm số gửi tiền lãi 50 + 3,5 = 53,5 (triệu đồng)

Toàn số tiền đ−ợc gọi gốc để tính lãi năm sau

Tổng số tiền có đ−ợc cuối năm thứ hai 53,5 + 53,5 7% = 57,245 (triệu đồng) Kiểu tính lãi nh trờn gi l lói kộp

Dới số dạng toán kiểu tính lÃi kép mà thùc tÕ chóng ta th−êng lùa chän

1 Gưi tiỊn mét lÇn

Ban đầu gửi vào ngân hàng số tiền a (đồng) Biết lãi suất m(%) /năm khơng rút lãi vào cuối năm Tính vốn lẫn lãi (Tn) sau n năm

Bài toán Một ng−ời gửi vào ngân hàng số tiền 10 000 000 đồng với lãi suất 7% /năm Hỏi sau năm ng−ời nhận đ−ợc tiền gốc lẫn lãi?

Lời giải Giả sử ban đầu ng−ời gửi a đồng vào ngân hàng với lãi suất m(%) năm Gọi Tn số tiền ng−ời nhận đ−ợc sau n năm

Sau năm, số tiền có T1 = a + ma = a(1 + m) Sau năm, số tiền có

T2= a(1 + m) + a(1 + m)m = a(1 + m)2

Tơng tự, sau n năm, số tiền

Tn = a(1 + m)n−1+ a(1 + m)n−1 m = a(1 + m)n áp dụng vào toán với a = 10 000 000 (đồng), m = 7%, n = số tiền ng−ời nhận đ−ợc T5 = 10 000 000.(1 + 7%)5

≈ 14 025 517 (đồng)

2 Gưi tiỊn hµng th¸ng

Hàng tháng gửi vào ngân hàng số tiền nh− a (đồng) Biết lãi suất m(%) /tháng không rút lãi vào cuối tháng Hỏi sau n tháng, ng−ời có tiền?

Bài toán Một ng−ời hàng tháng gửi 000 000 đồng vào ngân hàng để tiết kiệm cho tuổi với lãi suất 0,45% /tháng Hỏi 18 tuổi (sau 12 năm) ng−ời nhận đ−ợc tiền gốc lẫn lãi?

(9)

Ci th¸ng thø nhÊt sè tiỊn cã lµ T1 = a(1 + m)

Vì hàng tháng ng−ời tiếp tục gửi tiết kiệm a đồng nên số tiền gốc đầu tháng thứ hai

+ + = + +

= + − = + −

+ −

2

a(1 m) a a[(1 m) 1]

a a

[(1 m) 1] [(1 m) 1]

(1 m) m

Số tiền cuối tháng thứ hai

= + 2− +

2 a

T [(1 m) 1](1 m) m

Sè tiền gốc lẫn lÃi vào cuối tháng thứ n lµ

= + n− +

n a

T [(1 m) 1](1 m) m

áp dụng vào toán với a = 000 000 (đồng), m = 0,45%, n = 12 ì 12 = 144 số tiền ng−ời nhận đ−ợc

= + 144− +

12 1000 000

T [(1 0,45%) 1](1 0,45%)

0, 45%

≈ 202 894 858 (đồng)

3 Rút sổ tiết kiệm theo định kì

Số tiền ban đầu gửi vào ngân hàng a (đồng), lãi suất m(%) /tháng, tháng rút số tiền nh− T (đồng) Tính số tiền lại sau n tháng

Bài toán Bố bạn Minh gửi tiết kiệm cho bạn Minh 20 000 000 đồng vào ngân hàng với lãi suất 0,45% /tháng Nếu hàng tháng bạn Minh rút từ ngân hàng 900 000 (đồng) vào ngày ngân hàng tính lãi để sinh hoạt học tập Hỏi sau năm bạn Minh tiền ngân hàng?

Lời giải Gọi số tiền gửi ban đầu a (đồng), số tiền rút hàng tháng T (đồng), lãi suất m(%) /tháng, thời gian gửi n

Sau th¸ng sè tiỊn cã a + am = a(1 + m) = ak (víi k = + m)

Số tiền lại sau rót lµ ak − T

Sau tháng số tiền có ak T + (ak − T)m = (ak − T)(1 + m) = (ak T)k = ak2 Tk Số tiền lại sau rót lµ

− − = − + = −

2 2 k

ak Tk T ak T(k 1) ak T

k

Sau th¸ng sè tiÒn cã

⎛ − ⎞ − − = − ⎜ ⎟ ⎜ − ⎟ − ⎝ ⎠

2 k k k

ak T k ak T

k k

Số tiền lại sau rút

− −

− − = −

− −

3

3 k k k

ak T T ak T

k k

Sau n th¸ng sè tiền lại sau rút

− n

n k

ak T

k

Vậy sau năm (12 tháng), bạn Minh số tiền ngân hàng

+ + − − + − 12 12 20 000 000.(1 0,45%)

(1 0, 45%) 900 000

(1 0, 45%) ≈10 035 784 (đồng)

Bµi tËp tù luyÖn

Bài Một ng−ờigửi tiết kiệm 10 000 000 đồng vào ngân hàng khoảng thời gian 10 năm với lãi suất 5% /năm Hỏi ng−ời nhận đ−ợc số tiền nhiều hay ngân hàng trả lãi suất 4,2% /tháng?

Bài Một ng−ời muốn sau năm phải có 20 triệu đồng để mua xe Hỏi ng−ời phải gửi vào ngân hàng khoản tiền nh− hàng tháng bao nhiêu, biết lãi suất tiết kiệm 0,45% /tháng?

Bài Một ng−ời gửi vào ngân hàng 50 triệu đồng vòng năm với lãi suất 0,5% /tháng Hỏi cuối tháng, vào ngày ngân hàng tính lãi, ng−ời rút số tiền nh− để sau năm vừa hết số tiền gửi trên?

(10)

h−ớng dẫn giải đề THI imc 2019

võ quốc bá cẩn

Tr−êng Archimedes Academy, Q Cầu Giấy, Hà Nội

(Đề đăng TTT2 số 203) Trong này, trình bày lời giải

chi tit cho phn II (điền đáp số) phần III (tự luận)

D−ới đáp án phần I (trắc nghiệm) D; B; B; D;

5 D; B; A; C

D−íi lời giải chi tiết phần II phÇn III

9 (n 2)(n 1)n(n 1) 1+ + − + = (n2+ −n 2)(n2+ +n)

= (n2+n)2 −2(n2+ + =n) (n2+ −n 1)2 =| n2+ −n 1|

10 §iỊu kiƯn: x Đặt a= 2x 5+

=1 x

b ,

2 ph−ơng trình cho trở thành − + = + ⇔ − = − ⇔ − =1 1 a b

a b a b a b

a b b a ab

Từ ta có a = b ab = Nếu a = b, ta có

+ = − ⇔ = −

2x x

x

3

NÕu ab = 1, ta cã

+ −

⎛ ⎞⎛ ⎞= ⇔ + + =

⎜ ⎟⎜ ⎟

⎝ ⎠⎝ ⎠

⇔ = − = −

2x x

1 (x 1)(2x 1)

3

1

x 1; x

2

Vậy ph−ơng trình cho có tập nghiệm

⎧ ⎫

= −⎨ − ⎬

⎩ ⎭

1

S 1,

2

11 Rõ ràng đờng thẳng AB không song song với trục hoành nên phơng trình đờng thẳng AB có dạng y = ax + b Do đờng thẳng

AB qua hai điểm A(1, 1) B(2, 3) nên + = = − = − ⎨ + = − ⎩

a b

a , b

2a b 3

Do đ−ờng thẳng AB có ph−ơng trình = −4 −1

y x

3 3

Gọi d đ−ờng thẳng cần tìm Dựng tam giác ABC cho C nằm phía đ−ờng thẳng AB (tức ta phải có yC> −4xC−

3 3) th× C thuéc d

Do tam giác ABC nên BC2= CA2= AB2 Từ đó, ta có

− + + = + + −

= + + − − =

2 2

C C C C

2

(x 2) (y 3) (x 1) (y 1)

(2 1) ( 1) 25

Vì (xC2)2+(yC+3)2=(xC+1)2+(yC 1)2 nên 6xC8yC11 0,= hay xC= 8yC6+11 L¹i cã + 2+ − =

C C

(x 1) (y 1) 25 nªn + ⎛ ⎞ + + − = ⎜ ⎟ ⎝ ⎠ ⇔ + − = 2 C C C C 8y 11

1 (y 1) 25

6

100y 200y 575

Giải phơng trình này, ta đợc +

=

C 3

y

2 (t−¬ng øng,

+ =

C

x

(11)

hoặc yC= 3

2 (tơng øng,

− =

C x

2 ) Đối chiếu với điều kiện yC > −43xC −31, ta tìm đ−ợc tọa độ điểm C

⎛ + − + ⎞

⎜ ⎟

⎜ ⎟

⎝ ⎠

1 3

,

2

Do B(2, −3) vµ ⎛⎜⎜ + − + ⎞⎟⎟

⎝ ⎠

1 3

C ,

2 thuộc

d nên d không song song víi trơc tung

Từ đó, ph−ơng trình d có dạng y = mx + n với (m, n) nghiệm hệ ph−ơng trình

⎧ + + = − ⎧ ⎪ = ⎪ ⇔⎪ ⎨ + − + ⎨ + = + ⎪ ⎪ = − ⎩ ⎪⎩

48 25

2m n m

39

1 3

m n 213 50

n

2

39 Vậy phơng trình đờng thẳng d cần tìm y= 48 25 3+ x 213 50 3+

39 39

12 Do ph−ơng trình cho ln có nghiệm thực với a nên biệt thức Δ' ph−ơng trình phải ln khơng âm với a, tức ta phải có a2 + a − 2b ≥ với a, hay

+ ≥ ∀

a a

b,

2 a

Điều xảy b nhỏ giá trÞ nhá nhÊt cđa biĨu thøc

+ = a2 a

T

2 Ta thÊy

⎛ + ⎞ − − ⎜ ⎟ + ⎝ ⎠ = = ≥ = −

2 a 1

a a 4

T

2 2

Đẳng thức xảy a= −1

2 nªn min= −

T

8 Từ suy b≤ −1

8

13 Giải thiết toán đ−ợc viết lại thành (a − 5)2 + (b − 12)2 + (c − 13)2 = Từ suy a = 5, b = 12, c = 13

Vì 52 + 122 = 132 nên tam giác ABC vuông C, suy đ−ờng cao ứng với cạnh lớn tam giác ABC đ−ờng cao kẻ từ C Gọi hc độ dài đ−ờng cao

Ta cã SABC = 12chc = 12ab Suy hc = ab =5 12⋅ = 60

c 13 13

14.Từ đồ thị, ta suy k < điểm A(2, 0) thuộc đồ thị hàm số cho, nên 2k + b = ⇔b = −2k

Ta cã k(x − 3) + b > ⇔ k(x − 3) − 2k > ⇔ k(x − 5) > ⇔ x < (do k < 0)

Vậy tập tất số thực x thỏa mãn yêu cầu đề S=

{

x∈ |x<5

}

15 Gọi O tâm đ−ờng trịn cho Kí hiệu đỉnh hình vng nh− hình vẽ bên d−ới

Gọi B trung điểm AE C trung điểm DF O thuộc đờng thẳng BC

Gọi x độ dài cạnh hình vng nhỏ ta có AB= x

2 CD = x áp dụng định lý Pythagoras tam giác OAB (vuông B) OCD (vng C), ta có

= = + = +

= = + = +

2

2 2

2 2 2

x

1 OA AB BO BO ,

4

1 OD CD CO x CO

Nếu O thuộc tia đối tia CB OB = 2x + OC nên 1= x2 +OB2 = x2 +(2x OC)+

4

≥x2+(2x)2+OC2>x2+OC2=1

4 (m©u thuÉn)

Nếu O thuộc tia đối tia BC OC = 2x + OB nên x= 2+OC2= x2+(2x OB) +

>x2+OB2> x2 +OB2=1.

4 (m©u thuÉn)

(12)

⎧ + − = ⎪ ⎨ ⎪ + = ⎩ 2 2 x

(2x y)

x y

Trừ hai phơng trình theo vế, ta đợc

− =

1

x(13x 16y)

4 Suy =

13

y x

16 Thay vào phơng trình x2+y2=1, ta đợc

+⎜ ⎟ = ⇔ =

⎝ ⎠

2

2 13 16 17

x x x

16 85 (do x ) >

Vậy độ dài cạnh hình vng 16 17

85

16 Vì A C đối xứng với qua đ−ờng thẳng BD nên PA = PC

Suy PE + PC = PE + PA AE

Đẳng thức xảy vµ chØ P trïng víi Po lµ giao điểm AE BD

Do ú tổng PE + PC đạt giá trị nhỏ P trùng với P o

Qua E kẻ đ−ờng thẳng song song với BD cắt AB F tam giác BEF vng cân B Từ suy BE = BF = 2, AF =

=

EF 2 áp dụng định lí Thales tam giác AEF với P B // EF, ta đ−ợc o

⋅ ⋅

= ⇒ = = =

o

o

P B AB AB EF 2 10

P B

EF AF AF 7

VËy PE + PC nhá nhÊt th× PB P B= o =10 27 17 Trờng hợp 1: b < a

Đặt BG = x th× AG = a − x Do hai tam giác FAG FEH nên EH = AG = a x Mặt khác, hai tam giác GBC HDC nên DH = x

Từ suy ED = a − 2x

Vì AE + DE = AD nên b + (a − 2x) = a, hay b = 2x Nh vậy, trờng hợp này, ta có

= =

BG x

AE b

Tr−êng hỵp 2: a < b < 2a

Đặt BG = x AG = a − x Do hai tam giác FAG FEH nên EH = AG = a − x Mặt khác, hai tam giác GBC HDC nên DH = x Từ suy ED = 2x − a Vì AD + DE = AE nên a + (2x − a) = b Suy b = 2x Nh− vậy, tr−ờng hợp này, ta có BG= =x

AE b

Tóm lại, hai tr−ờng hợp ta có =

BG

AE

18 Ta cã = + = − − ≤

+ +

2

2

x 1 (x 1)

y

2

x 2(x 3)

Đẳng thức xảy x = nªn =

=max y

a

2 T−¬ng tù, ta cịng cã

+ +

= = − + ≥ −

+ +

2

2

x 1 (x 3)

y

6

x 6(x 3)

Đẳng thức xảy x = −3 nªn −

=min y=

b

6

VËy a b+ = − =1 1

(13)

Ai thủ phạm

gây án?

Trần Phơng Nam (ghi l¹i)

ơi sang nhà thám tử Sê Lốc Cốc để cảm ơn anh kể nghe vụ án có liên quan tới số điện thoại nạn nhân có nhiều bạn đọc nhỏ tạp chí Tốn Tuổi thơ hứng thú phá án thành công

Lần này, đề nghị thám tử kể cho nghe vụ án mà nhờ kiến thức môn học khác Tốn để tìm thủ phạm

Sª Lèc Cèc nhìn cời:

- Cỏc bn nh ca Toỏn Tuổi thơ thật đấy! Phá án vụ cách áp dụng kiến thức mơn Tốn vào sống Bây thử xem bạn nhỏ cú gii mụn Hoỏ hc khụng nhộ!

Tôi tròn xoe mắt:

- Vụ án có liên quan tới kiến thức môn Hoá? Sê Lốc Cốc chậm rÃi:

- Đúng thế! Một hôm cảnh sát tr−ởng gọi đến gấp tr−ờng vụ án xảy

phịng thí nghiệm Hố học Viện nghiên cứu Khi tơi tới nơi nạn nhân nằm gục nửa ng−ời bàn làm việc Tôi đeo găng nhẹ lật nạn nhân lên chút thấy tờ giấy loang lổ máu với tên hai hoá chất ghi từ bút nằm cạnh Tên hoá chất ghi tờ giấy là: Lithium

Sodium

Tôi hỏi cảnh sát tr−ởng: “Ai báo tin cho ngài?” Cảnh sát tr−ởng kể: “Nhân viên bảo vệ thấy muộn mà giáo s− ch−a nên vào phịng thí nghiệm Khi thấy cảnh t−ợng điện báo cho cảnh sát địa ph−ơng Cảnh sát địa phng bỏo tụi.

(14)

danh sách mà bảo vệ cung cấp, kể bảo vệ có ng−êi danh s¸ch:

Piter Lina Leo Lotus Julia

Tơi c−ời nói với ơng ta: Tơi cần địa ng−ời số đến bắt kẻ thủ phạm kẻo tên “cao chạy, xa bay”

Nghe đến đấy, ngạc nhiên:

- Tại thám tử lại biết đ−ợc đích danh thủ phạm?

Th¸m tư c−êi:

- Giáo s− kịp ghi lại… tên thủ phạm Nhớ kiến thức Hoá học từ thời học nên chẳng khó khăn để biết tên kẻ giết ng−ời

Tôi vốn dốt môn Hoá nên ngơ ngác Sê Lốc Cốc vỗ vai:

- Đăng lên Toán Tuổi thơ Các bạn nhỏ biết thủ phạm!

No cỏc bn nh yờu chuyờn mục tay nhé! Các bạn tra cứu lại kiến thức Hoá học để nhanh chóng biết tên thủ phạm nh− thám tử Sê Lốc Cốc nhé!

(TTT2 sè 202)

Nh÷ng sè

trên điện

thoại nạn nhân

Trong vụ án này, manh mối để tìm thủ phạm điện thoại nạn nhân với dãy số 2-4-8-10 hình điện thoại Thoạt nhìn dãy số đó, nghĩ tới quy luật dãy số, dãy số có liên quan tới chữ Nh−ng “chìa khóa” để giải mã “bí ẩn” lại nằm đồng hồ treo t−ờng Chúng ta để ý tới bốn số 2, 4, 8, 10 đồng hồ, nối chúng lại với theo thứ tự tạo thành hình chữ nhật Kết hợp với hình xăm cánh tay kẻ tình nghi, ta khng nh chc chn

rằng: Ngời có vết xăm hình chữ nhật thủ phạm vụ trọng án

Nhận xét Kì có nhiều bạn tham gia phá án Thật vui tất bạn tài tình không thám tử Sê Lốc Cốc Phần quà đợc gửi tới bạn sau: Dơng Nh Quỳnh, 7A, THCS Lý Tự Trọng, Bình Xuyên,

Vĩnh Phúc; Lu Thùy Dơng, 8A3, THCS Trần Đăng Ninh, TP Nam Định, Nam Định;

Đinh Thúy Vân, 8B, THCS Tân Bình, TP Tam Điệp, Ninh Bình; Lê Phú Quang, 7D, THCS Nhữ Bá Sỹ, Hoằng Hóa, Thanh Hóa; Trần Minh Hoàng, 7E, THCS Nguyễn TrÃi, Nghi Xuân, Hà Tĩnh

Cỏc t nhớ sau cng đáng khen: Nguyễn Kim Dung, 9A5, THCS Cầu Giấy, Q Cầu Giấy, Hà Nội; L−u Thành Đạt, 7A1, THCS Nguyễn Đăng Đạo, TP Bắc Ninh; Bùi Bá Minh Đức, 6A, Lê H−ơng Thảo, 6C,

NguyÔn Th−êng Phong, 7A, THCS Hàn Thuyên, Lơng Tài, Bắc Ninh; Trần Trung Phúc, 8A4, THCS Ngô Gia Tự, Hồng Bàng,

Hải Phòng; Nguyễn Huy Hoàng Sơn, 6A2,

Nguyễn Long An, 7A1, THCS Yên Lạc, Yên Lạc; Bạch Thái Sơn, 7A1, THCS Vĩnh Yên, TP Vĩnh Yên, Vĩnh Phúc; Vũ Đức Đạt, 7C,

Hoàng Minh Vũ, 7D, THCS Văn Lang, TP Việt Trì; Phạm Minh Châu, Đỗ Ngọc Tiến, 7A3, THCS Lâm Thao, Lâm Thao, Phú Thọ;

Phạm Hoàng Nhật Minh, 6A, Đoàn Nguyên Vũ, Lu Ngọc Diệp, 6B, Đoàn Xuân Bảng, 7D, THCS Nguyễn Hiền, Nam Trực, Nam Định; Phan Thùy Trâm, Ngô Bích Loan, 7C, THCS Bạch Liêu, Yên Thành, Nghệ An; Lê Thị Phơng Thảo, 8A, Lê Thị Diệu Thúy, 9A, THCS Bình Thịnh, Đức Thọ, Hà TÜnh

(15)

cã tån t¹i tam giác không?

Bi toỏn Trong tam giỏc ABC v đ−ờng cao AH, trung tuyến BG phân giác CK, chúng cắt tạo thành tam giác DEF (nh− hình vẽ) Hỏi có tam giác ABC để tam giác DEF tam giác hay khơng?

Ngun xu©n bình (Nhà xuất Giáo dục Việt Nam) (TTT2 sè 202)

GãC SóT LíN NHÊT

Lời giải Vị trí tốt để sút bóng vào l−ới vị trí E đ−ờng thẳng d cho đ−ờng tròn (ABE) tiếp xúc với d

Ta chứng minh góc nhìn từ E đến AB khơng nhỏ góc nhìn từ điểm M tùy ý d n AB

Thật vậy, gọi M điểm tùy ý d, đờng thẳng MA cắt đờng tròn (ABE) điểm C Ta có

ACB>AMB (góc tam giác BMC) Mà ACB=AEB (cùng chắn cung AB

đờng tròn (ABE)) Suy AEB>AMB

Vy im E đ−ờng thẳng d có góc sút lớn đến khung thành AB

Nhận xét Rất đáng tiếc kì khơng bạn có lời giải

(16)

một số ph

ơng pháp

tìm cực trị hình học

trơng quang an

Nghĩa Thắng, T Nghĩa, Quảng NgÃi Bài viết xin giới thiệu số

phơng pháp tìm cực trị thờng gặp toán hình học phẳng

1 VËn dơng quan hƯ gi÷a độ dài đờng xiên đờng vuông góc

Bi toỏn Cho hình vng ABCD Trên cạnh AB, BC, CD, DA ta lấy theo thứ tự điểm E, F, G, H cho AE = BF = CG = DH Xác định vị trí điểm E, F, G, H cho tứ giác EFGH có chu vi nhỏ Lời giải

DÔ thÊy

ΔHAE = ΔEBF = ΔFCG = ΔGDH (c.g.c) Suy HE = EF = FG = GH vµ AHE BEF =

⇒BEF AEH AHE AEH 90 + = + = o Từ ta có tứ giác EFGH hình vuông Gọi O giao điểm AC EG

Tø gi¸c AECG cã AE = CG, AE // CG nên AECG hình bình hành

Suy O trung điểm AC EG, O tâm hai hình vng ABCD EFGH

Vì ΔHOE vng cân O nên HE= 2OE Chu vi tứ giác EFGH 4HE 2OE = Do chu vi EFGH nhỏ

⇔ OE nhá nhÊt

KỴ OK⊥AB(K AB) ∈

Theo quan hệ độ dài đ−ờng vng góc đ−ờng xiên OE ≥ OK (độ dài OK không đổi) nên OE = OK ⇔E ≡ K

Do chu vi tứ giác EFGH nhỏ E, F, G, H lần l−ợt trung điểm AB, BC, CD, DA

Bài tốn Cho đoạn thẳng AB có độ dài 2a Vẽ phía AB tia Ax By vng góc với AB Qua trung điểm M AB có hai đ−ờng thẳng thay đổi ln vng góc với cắt Ax, By theo thứ tự C, D Xác định vị trí điểm C, D cho tam giác MCD có diện tích nhỏ nht

Lời giải Gọi K giao điểm CM vµ DB ΔMAC = ΔMBK (g.c.g) ⇒ MC = MK

DCK có đờng cao DM trung tuyến nên DCK tam giác cân D

Suy CDM KDM.=

KỴ MH CD(H CD) ⊥

Do M thuộc tia phân giác góc CDK nªn MH = MB = a

Ta cã

= ≥ = =

MCD 1

S CD.MH AB.MH 2a.a a

(17)

Đẳng thức xảy CD ⊥ Ax,

= o = o

AMC 45 , BMD 45

Do tam giác MCD có diện tích nhỏ điểm C, D đ−ợc xác định Ax, By cho AC = BD = a

Bài toán Cho tam giác ABC có góc B góc tù, điểm D di chuyển cạnh BC Xác định vị trí điểm D cho tổng khoảng cách từ B từ C đến đ−ờng thẳng AD có giá trị lớn

Lêi gi¶i

Gäi E, F lần lợt hình chiếu B, C AD Ta cã SABD+ SACD= SABC hay

+ = ABC

1

.AD.BE AD.CF S

2

Suy BE + CF = 2SABC AD

Do BE + CF lớn đ−ờng xiên AD nhỏ hay hình chiếu HD AD BC nhỏ

Vì HD HB (do ABD 90> o) nên HD = HB vµ chØ D ≡ B

Vậy D trùng B tổng khoảng cách từ B từ C đến AD có giá trị lớn Quan hệ độ dài đoạn thng v

di đờng gấp khúc

Bài to¸n Cho tam gi¸c ABC nhän Dùng tam gi¸c MNP néi tiÕp tam gi¸c ABC (M thuéc AB, N thuéc BC, P thuéc AC) cho tam MNP cã chu vi nhá nhÊt

Lời giải Dựng điểm E, F đối xứng với N lần l−ợt qua AB, AC (Hình a)

Chu vi tam gi¸c MNP lµ

NM + MP + PN = EM + MP + PF EF Ta cần xét th× EF nhá nhÊt

(H×nh a) Ta cã EAF 2A= 1+2A2=2BAC

Vì ΔEAF tam giác cân có góc đỉnh khơng đổi nên cạnh đáy EF nhỏ cạnh bên AE nhỏ hay AN nhỏ nhất, tức N hình chiếu vng góc A cạnh BC (Hình b)

Xét tam giác NMP có MB đờng phân giác góc NMP, PC đờng phân giác góc NPM mà MB cắt PC A nên NA tia phân giác góc MNP

(Hình b)

Vì AN NC nên NC đờng phân giác góc MNP, PC đờng phân giác góc MPN

M NC cắt PC C nên MC tia phân giác góc NMP, từ suy MB ⊥ MC T−ơng tự PC ⊥ PB

(18)

Lời giải Dựng hình bình hành BNMB’ BB’ = MN = a (không đổi) nên B’ cố định

Gọi A’ điểm đối xứng A qua đ−ờng thẳng d A’ cố định A’M = AM

Ta có AM + MN + NB =A’M + MN + MB’ = (A’M + MB’) + MN ≥ A’B’ + a (không đổi) Đẳng thức xảy M trùng với M1 giao điểm A’B’ với đ−ờng thẳng d, N trùng với N1 (M1N1 = a, N1 ∈ d)

3 Quan hÖ di dây cung đờng kính đờng tròn

Bài tốn Cho nửa đ−ờng trịn (O; R) đ−ờng kính AB M điểm di động nửa đ−ờng tròn Qua M kẻ tiếp tuyến với nửa đ−ờng trịn Gọi D, C lần l−ợt hình chiếu A; B tiếp tuyến Xác định vị trí điểm M để diện tích tứ giác ABCD có giá trị lớn

Lêi gi¶i Ta cã AD ⊥ DC, BC ⊥ DC

Suy AD // BC nên ABCD hình thang vuông

Vì OM DC nên OM // AD O trung điểm AB nên OM đờng trung bình hình thang ABCD

Do SABCD = AD BC+ DC OM.DC.=

VÏ AE ⊥ BC ⇒ ADC DCE AEC 90= = = o nên tứ giác ADCE hình chữ nhật

DC = EA

Lại có AEB 90 nên E thuộc nửa đờng = o tròn (O) đờng kính AB

T ú ta có

= ≤ = =

ABCD

S AE.OM AB.OM 2R.R 2R

(không đổi)

Đẳng thức xảy AE đ−ờng kính (O; R) OM ⊥ AB O

M điểm cung AB

Bài tốn Cho đ−ờng trịn (O; R), BC dây cung cố định (BC≠2R) A điểm chuyển động cung lớn BC Xác định vị trí A để chu vi tam giác ABC lớn Lời giải Trên tia đối tia AB lấy D cho AD = AC Ta có ΔADC cân A nên

=BAC =1 = α

BDC s® BC

2 (không đổi) Mà BC cố định, suy D thuộc cung chứa góc α dựng đoạn thẳng BC

Gäi PABC lµ chu vi cđa tam gi¸c ABC

Ta có PABC = AB + AC + BC (BC không đổi) lớn

⇔ AB + AC = AB + AD = BD đạt giá trị lớn hay BD đ−ờng kính cung chứa góc nói Khi

= o = + = +

BCD 90 ACB ACD ABC ADC Mà ACD ADC (vì AC = AD) =

Do ABC ACB ⇔ sđ AB= = sđ AC

Suy A điểm cđa cung lín BC

4 Sử dụng bất đẳng thức đại số

(19)

Lời giải Tam giác AMB có AMB 90= onên theo định lí Pythagoras ta có

+ = =

2 2

MA MB AB 4R

áp dụng bất đẳng thức Bunyakovsky ta có

+ = +

≤ + 2+ = =

MA 3.MB | MA 3.MB |

(1 3)(MA MB ) 4.4R 4R Đẳng thức xảy chØ

= = o ⇔ = o

MA

tan30 ABM 30

MB

Vậy MA+ 3MB đạt giá trị lớn M thuộc cung AB cho sđMA 60 = o

Bài toán Cho điểm M di động đoạn thẳng AB có độ dài a khơng đổi Trên nửa mặt phẳng bờ chứa AB, vẽ tam giác AMC BMD Xác định vị trí M để tổng diện tích tam giác AMC v BMD l nh nht

Lời giải Đặt MA = x, MB = y Ta cã

= =

AMC x BMD y

S ,S

4

Do

+ = +

+

≥ =

2 AMC BMD

2

2

S S (x y )

4 (x y)

a

4

Đẳng thức xảy x = y M trung điểm AB

VËy SAMC + SBMD nhá nhÊt b»ng 3a2

8 M trung điểm đoạn thẳng AB

Bi tốn 10 Cho hình vng ABCD có cạnh 12 cm, E trung điểm CD, điểm F thuộc cạnh BC cho CF = cm Các điểm G H theo thứ tự di chuyển cạnh AB AD cho GH // EF Xác định vị trí điểm G cho tứ giác EFGH có diện tích lớn Tính diện tích lớn nht ú Li gii

Đặt BG = x th× AG = 12 − x

V× EF // GH nªn ΔGAH ΔECF (g.g) Suy

= ⇔ = ⇒ = −

AH AG AH 12 x

AH x

CF CE

⇒DH AD AH 4= − = + 2x

3 Ta cã SEFGH = SABCD − SAGH − SBFG − SCEF − SDHE

⎛ ⎞

= − ⎜ − ⎟ − − − − +

⎝ ⎠

1

144 x (12 x) 4x 12 (12 2x)

2

=144 48 8x− + −1x2−6x 24−

= −1x2+2x 72+ = −1(x 3)− 2+75 75.≤

3

Đẳng thức xảy x = hay điểm G nằm cạnh AB cho BG = cm VËy tø gi¸c EFGH cã diƯn tÝch lín nhÊt b»ng 75 cm2

Bài tập tự luyện

Bài Cho tam giác ABC cân (AB = AC) Lấy điểm D cạnh BC (D kh¸c B,C) Gäi

1

r ,r lần l−ợt bán kính đ−ờng trịn nội tiếp tam giác ABD ACD Xác định vị trí D để tích r1r2 đạt giá trị lớn

(20)

Một số ứng dụng

bất đẳng thức Minkowski

Bïi Minh Trung

Tr−êng THCS Mỹ Long Nam, Cầu Ngang, Trà Vinh

Bt ng thức Minkowski: Cho hai dãy số thực (a1, a2, ., an) (b1, b2, ., bn) ta

lu«n cã a12+b12 + a22+b22 + + an2+bn2

2

1 n n

(a a a ) (b b b )

≥ + + + + + + +

Nếu b1, b2, , bn khác đẳng thức xảy

khi 1= = = n n

a a a

b b b Bất đẳng thức chứng minh hình học nhờ cơng thức tính khoảng cách hai điểm mặt phẳng tọa độ D−ới số ứng dụng bất đẳng thức

1 Sử dụng bất đẳng thức Minkowski để giải ph−ơng trỡnh

Bài toán Giải phơng trình

2

x +12x 61+ + x −14x 113+ = 338 (1)

Lời giải ĐKXĐ

+ + ≥ ⎧ + + ≥ ⎪ ⇔⎪ ⎨ ⎨ − + ≥ − + ≥ ⎪ ⎪ ⎩ ⎩ 2 2

x 12x 61 (x 6) 25

x 14x 113 (x 7) 64

Do ph−ơng trình xác định với số thực x Ph−ơng trình cho t−ơng đ−ơng với

+ 2+ + − 2+ =

(x 6) (7 x) 338

áp dụng bất đẳng thức Minkowski ta đ−ợc VT(1) = (x 6)+ 2+52 + (7 x)− 2+8

≥ (x x)+ + − 2+ +(5 8)2 = 338= VP(1) Đẳng thức xảy (x 6).8 5(7 x) + = −

⇔ = −x

Do x= −1 nghiệm ph−ơng trỡnh

Bài toán Giải phơng trình

+ + − +

= − +

2

2

8x 16x 10 2x 4x

7 x 2x (1)

Lời giải ĐKXĐ

⎧ − + ≥ ⎧ − + ≥ ⎪ ⎪ ⎪ − + ≥ ⇔⎪ − + ≥ ⎨ ⎨ ⎪ ⎪ − + ≥ − ≤ ⎪ ⎪ ⎩ ⎩ 2 2 2

8x 16x 10 8(x 1)

2x 4x 2(x 1)

7 x 2x (x 1)

⇔ −1 2 x 2 +

Phơng trình (1) tơng ®−¬ng víi

− + − + − +

= − −

2 2

2

(2x 1) (3 2x) (2 x) x

8 (x 1) (2)

áp dụng bất đẳng thức Minkowski ta đ−ợc

2 2

2

2

(2x 1) (3 2x) (2 x) x

(2x x) (3 2x x)

2x 4x 10 2(x 1) 8 (3)

− + − + − +

≥ − + − + − +

= − + = − + ≥

Tõ (2) vµ (3) ta cã

− − ≥ ⇔ − − ≥

⇔ − ≤

2

2

8 (x 1) 8 (x 1)

(x 1)

Đẳng thức xảy − =

⎧ ⎨

− = − −

x

(2x 1)x (3 2x)(2 x)

2

x

x

2x x 2x 7x

= ⎧⎪

⇔⎨ ⇔ =

− = − +

⎪⎩

(Tháa m·n §KX§)

(21)

2 Sử dụng bất đẳng thức Minkowski để giải hệ ph−ơng trỡnh

Bài toán Giải hệ phơng trình

⎧ + + = ⎪ ⎨ ⎪ − + + − + = ⎩ 2 2

x xy y 12

x x y y

Lời giải áp dụng bất đẳng thức Minkowski ta có

− + + − +

2

x x y y

⎛ ⎞ ⎛ ⎞ ⎛ ⎞ ⎛ ⎞ = ⎜ − ⎟ +⎜⎜ ⎟⎟ + ⎜ − ⎟ +⎜⎜ ⎟⎟ ⎝ ⎠ ⎝ ⎠ ⎝ ⎠ ⎝ ⎠ 2 2

x y

1 x y

2 2

⎛ ⎞ ⎛ ⎞ ≥ ⎜ − + − ⎟ +⎜⎜ + ⎟⎟ ⎝ ⎠ ⎝ ⎠ 2

x y 3x 3y

1

2 2

= x2+xy y+ = 12 = Đẳng thức xảy x = y =

VËy nghiƯm cđa hƯ lµ (x ; y) = (2 ; 2)

Bài toán Giải hệ phơng trình + + + = − − = ⎪⎩ 2 2

1

x y

2

x y

x y 2

Lời giải ĐKXĐ x 2; y ≥ ≥

Ta cã x+ = + +1 x 3x ≥2 x + 3.2=5

x x 4 x

T−¬ng tù y+ ≥1

y

áp dụng bất đẳng thức Minkowski ta có

⎛ ⎞ ⎛ ⎞ + + + ≥ ⎜ + ⎟ +⎜ + ⎟ ⎝ ⎠ ⎝ ⎠ ⎛ ⎞ ⎛ ⎞ ≥ ⎜ ⎟ +⎜ ⎟ = ⎝ ⎠ ⎝ ⎠ 2 2 2 2

1 1

x y x y

x y

x y

5 5

2 2

Đẳng thức xảy x = y =

Thay vµo phơng trình thứ hai thấy thỏa mÃn

Vậy nghiệm cđa hƯ lµ (x ; y) = (2 ; 2)

3 Sử dụng bất đẳng thức Minkowski để chng minh bt ng thc

Bài toán Cho x, y, z số thực dơng Chứng minh r»ng

− + + − + ≥ + +

2 2 2

x xy y y yz z z zx x

Lời giải áp dụng bất đẳng thức Minkowski ta có x2−xy y+ + y2−yz z+

⎛ ⎞ ⎛ ⎞ ⎛ ⎞ ⎛ ⎞

= ⎜⎜ ⎟⎟ +⎜ − ⎟ + ⎜⎜ ⎟⎟ +⎜ − ⎟

⎝ ⎠ ⎝ ⎠

⎝ ⎠ ⎝ ⎠

2 2 2

3 x z

x y z y

2 2

⎛ ⎞ ⎛ ⎞ ≥ ⎜⎜ + ⎟⎟ +⎜ − ⎟ ⎝ ⎠ ⎝ ⎠ = + + 2 2

3 x z

x z

2 2

z zx x

Bài toán Cho số thực dơng a, b, c tháa m·n ab + bc + ca = abc Chøng minh r»ng

+ + + + + ≥

2 2 2

b 2a c 2b a 2c

3

ab bc ca

Lêi gi¶i Tõ gi¶ thiÕt ta suy 1+ + =1 1

a b c

áp dụng bất đẳng thức Minkowski ta có

2 2 2

2 2 2 2

2

2 2

b 2a c 2b a 2c

ab bc ca

1 2

a b b c c a

1 2

a b b c c a

1 1 2

a b c a b c

1 1

3

a b c

3.1 + + + + + = + + + + + ⎛ ⎞ ⎛ ⎞ ≥ ⎜ + ⎟ +⎜⎜ + ⎟⎟ + + ⎝ ⎠ ⎝ ⎠ ⎛ ⎞ ⎛ ⎞ ≥ ⎜ + + ⎟ +⎜⎜ + + ⎟⎟ ⎝ ⎠ ⎝ ⎠ ⎛ ⎞ = ⎜ + + ⎟ ⎝ ⎠ = =

Đẳng thức xảy a = b = c =

Bài toán Cho x, y, z số thực dơng x y z Chøng minh r»ng + + ≤

= 2+ 12 + 2+ 12 + 2+ 12 ≥

P x y z 82

(22)

Lời giải áp dụng bất đẳng thức Minkowski ta có = + + + + + ⎛ ⎞ ≥ + + +⎜ + + ⎟ ⎝ ⎠

2 2

2 2

2

1 1

P x y z

x y z

1 1

(x y z) (1)

x y z

Mặt khác áp dụng bất đẳng thức AM -GM ta có

⎛ ⎞

+ + +⎜ + + ⎟

⎝ ⎠

2 1

(x y z)

x y z

⎛ ⎞

= + + +⎜ + + ⎟ − + +

⎝ ⎠

2

2 1

81(x y z) 80(x y z)

x y z

⎛ ⎞

≥ + + ⎜ + + ⎟ −

⎝ ⎠

2 1

2 81.(x y z) 80.1

x y z

⎛ ⎞

= + + ⎜ + + ⎟−

⎝ ⎠

≥ − =

1 1

2.9(x y z) 80

x y z

2.9.9 80 82 Thay vµo (1) ta ®−ỵc

= 2+ 12 + 2+ 12 + 2+ 12 ≥

P x y z 82

x y z

Đẳng thức xảy x y z= = =

4 Sử dụng bất đẳng thức Minkowski để tìm cực trị biểu thc i s

Bài toán Tìm giá trị nhá nhÊt cđa biĨu thøc P 9x= 2−6x 4x+ + 2+4x +

Lời giải áp dụng bất đẳng thức Minkowski ta có P 9x= 2−6x 4x+ + 2+4x +

= (2 6x)− 2+22 + (6x 3)+ 2+32 ≥ (2 6x 6x 3)− + + 2+ +(2 3)2 =5 Đẳng thức xảy 6x x

6x 3

− = ⇔ =

+ VËy Pmin=5 x =

Bµi toán Cho a, b, c số thực Tìm giá trị nhỏ biểu thức

2 2 2

P= a + −(1 b) + b + −(1 c) + c + −(1 a)

Lời giải áp dụng bất đẳng thức Minkowski ta có

= + − + + − + + −

≥ + + + − − −

2 2 2

2

P a (1 b) b (1 c) c (1 a)

(a b c) (3 a b c) Mặt khác ta có

+ + + − − − = + + − + + + ⎛ ⎞ = ⎜ + + − ⎟ + ≥ ⎝ ⎠ 2 2

(a b c) (3 a b c) 2(a b c) 6(a b c)

3 9

2 a b c

2 2

Đẳng thức xảy a b c= = = VËy Pmin= 22 a b c= = = 21

Bài toán 10 Cho a, b số thùc tháa m·n (a 2)(b 2)+ + = 25

4 Tìm giá trị nhỏ biểu thức F= 1 a+ + 1 b +

Lêi gi¶i Theo gi¶ thiÕt ta cã

+ + = 25⇔ + + =

(a 2)(b 2) ab 2a 2b

4

Ta l¹i cã

2

2

a b 1

ab; 2a 2a; 2b 2b

2 2

+ ≥ + ≥ + ≥

Cộng vế với vế bất đẳng thức ta có

+ + + + ≥ + + = ⇒ + ≥ ⇒ + ≥ 2 2 2 2

a b

2a 2b ab 2a 2b

2

5a 5b

a b

2

áp dụng bất đẳng thức Minkowski ta có

= + + + = + + +

⎛ ⎞

≥ + + + ≥ +⎜ ⎟ =

⎝ ⎠

4 2 2 2

2 2

F a b (a ) (b )

1 17

(1 1) (a b )

2

Đẳng thức x¶y a b= = VËy Fmin 17

2

(23)

Trận đấu thứ trăm sáu m−ơi CHíN

Ng−ời thách đấu: Nguyễn Minh Hà, Hà Nội

Bài toán thách đấu: Cho nửa đ−ờng trịn tâm O, đ−ờng kính AB Hai điểm C, D chạy nửa đ−ờng tròn cho COD 90= o

<

AOC AOD Tìm vị trí C, D cho tích AD.BC đạt giá trị ln nht

Thời hạn: Trớc ngày 08.3.2019 theo dÊu b−u ®iƯn

Trận đấu thứ trăm sáu m−ơi bảy

(TTT2 số 202) Đề Có n đội bóng tham gia giải thi

đấu bóng đá (n ∈ N, n ≥ 2) Giải đấu tổ chức theo thể thức thi đấu vòng tròn l−ợt Cách tính điểm trận đấu nh− sau: Đội thắng đ−ợc điểm, đội thua đ−ợc điểm, trận đấu có kết hịa đội đ−ợc điểm Khi kết thúc giải khơng có hai đội có điểm Hỏi đội đứng đầu đội xếp cuối điểm?

Lời giải Kí hiệu điểm đội thứ i, khơng tính tổng quát ta giả sử a1< a2 < < an

Vì số tự nhiên

Tn = an− a1= (an − an−1) + (an−1 − an−2) + + (a2− a1) ≥ n −

Ta kí hiệu aTb đội a thắng đội b, aHb đội a hòa đội b

Với n = 2: Có trận đấu 2T1 ⇒ a1= 0, a2= ⇒ T2=

Víi n = 3: XÐt c¸c khả a3 a2 = ⇒ T3 ≥ 3, T3 = nÕu 3T1, 3H2, 2H1 (a1= 1, a2= 2, a3= 4)

Víi n = 4: T4 = VÝ dô 4T3, 4H2, 4H1, 3H2, 3T1, 2H1 (a1= 2, a2= 3, a3= 4, a4= 5)

Giả sử n ≥ có kết đấu cho n đội mà a1= n − 2, a2= n − 1, , an= 2n − Bây ta có đội thứ n + với kết thi đấu: (n + 1)Tn, (n − 1)T(n + 1), (n − 2)T(n + 1),

(n + 1)T(n − 3), (n − 4)T(n + 1), (n − 5)T(n + 1),

• Nếu n = 3k điểm đội thứ 1, 2, 3, , n + t−ơng ứng n − 1, n + 2, n, n + 4, n + 5, n + 3, , 2n − 2, 2n − 1, 2n − 3, n + kết đội thứ (n + 1) với ba đội 1, 2, (n + 1)T3, 2T(n + 1), 1H(n + 1)

• Nếu n = 3k + 1, kết t−ơng ứng n − 1, n + 2, n + 3, n + 1, n + 5, , 2n − 2, 2n − 1, 2n − 3, n kết đội thứ (n + 1) với đội (n + 1)H1

• Nếu n = 3k + 2, kết t−ơng ứng n + 1, n, n + 3, n + 4, n + 2, , 2n − 2, 2n − 1, 2n − 3, n − kết đội thứ (n + 1) với hai đội 1T(n + 1), 2H(n + 1)

Cả ba tr−ờng hợp, điểm (n + 1) đội (n + 1) số nguyên d−ơng liên tiếp n − 1, n, n + 1, , 2n −

Nh n giá trị nhỏ Tn n 1, giá trị nhá nhÊt cña T3 b»ng 3, T2 chØ cã mét giá trị

Nhn xột õy l bi tốn tổ hợp hay khó, phần học sinh Việt Nam hay bị nhiều điểm kì thi học sinh giỏi quốc tế Có võ sĩ tham gia trận đấu, nh−ng tiếc khơng có võ sĩ giải toán Rất mong trận đấu sau có võ sĩ đăng quang trận đấu hấp dẫn nh−

(24)

Kì 48

HÃy thay chữ chữ số, biết chữ khác biểu diễn chữ số khác khác

MUA XUAN 2020.

+

=

Lª Anh tuÊn

(Qnh L−u, NghƯ An)

K× 46

(TTT2 sè 202)

Lời giải Do TOAN TOAN TUOITHOì = 9999 999 4000 4000

< < ×

Do T 1, 2, Mặt khác, T =

TOAN TOAN 2000 2000 000 000.ì > ì = Khi

TOAN TOAN 2UOI2HO 000 000ì = < (vô lí) Tơng tự, nÕu T = th×

TOAN TOAN 3000 3000 000 000.ì > ì = Khi

× = <

TOAN TOAN 3UOI3HO 000 000 (vô lí) Do T

Vì N ì N = O O N, O ≠ nªn N chØ cã thĨ b»ng 2, 3, 4, 7, O tơng ứng 4, 9, 6, 9,

NÕu O ≥ th×

TUOITHO TOAN TOAN 1500 1500 250000

= ×

≥ × =

⇒T ≥ (vô lí T = 1) Suy O chØ cã thĨ b»ng Víi O = th× N =

Khi A = A ≥ (vì A ≠ T = 1, A ≠ N

= 2, A ≠ O = 4)

Ta cã TOAN TOAN 14A2 14A2× = ×

1432 1432 050624,

ì = vô lí T = Vậy toán nghiệm

Nhận xét Có nhiều bạn gửi lời giải tòa soạn Tuy nhiên đa số bạn lập luận ch−a thực gãy gọn Nhiều bạn đ−a lời giải vắn tắt, số bạn lại giải q dài dịng Các bạn sau có lời giải đ−ợc th−ởng kì này: Đồn Minh Đức, 7A, THCS Hoàng Xuân Hãn, Đức Thọ; Trần Minh Hoàng, 7E, THCS Nguyễn Trãi, Nghi Xuân, Hà Tĩnh; Nguyễn Mạnh Hùng, 9A3, THCS Giấy Phong Châu, Phù Ninh; Đỗ Ngọc Tiến, 7A3, THCS Lâm Thao, Lâm Thao, Phú Thọ; Trần Trung Phúc, 8A4, THCS Ngô Gia Tự, Hồng Bàng,

Hải Phòng

(25)

thi kim tra chất l

ợng học sinh giỏi lớp 6

huyện ch

ơng mỹ, TP Hà Nội

Năm học 2018 - 2019 Môn thi: Toán

Thời gian làm bài: 150 phút Bài 1.(4,0 điểm)

1)Cho biểu thức

⎛ − ⎞ ⎜ ⎟ ⎝ ⎠

=

⎛ − ⎞ ⎜ ⎟ ⎝ ⎠

2 1,08 :

25 A

5

6 3,25

9 17

⎛ ⎞ ⎜ ⎟ ⎝ ⎠

= +

4 0,8 : 1,25

5

B (1,2.0,5) : 0,8

0,64 17 TÝnh A + 3B

2) Cho S= + + + +

3.7 7.11 11.15 2019.2023 So sánh S với 504

6068 Bài (4,0 điểm)

Tìm số nguyên x, y thỏa m·n:

1) (2x − 1)2 : = 49; 2) 3x+ 3x+2= 810;

3) 2x − =2 1;

3 y 4) 80 x; 56 x x Bài 3.(3,0 điểm)

1) Cho A = + 52+ 53+ + 52019 Chøng tá r»ng 4A + lµ sè chÝnh ph−¬ng

2) Chøng tá r»ng víi mäi sè tự nhiên n 3n + 5n + hai số nguyên tố Bài 4.(2,0 ®iĨm)

1) Cho ph©n sè = −

2019

P

x 2020 Tìm số nguyên x để P có giá trị lớn Tìm giá trị lớn 2) Hãy chia số 36 thành ba số a, b, c cho a =3

b vµ = b

c Bµi 5.(6,0 điểm)

1) Cho điểm O nằm đờng thẳng xy Trên tia Ox lấy điểm M cho OM = cm Trên tia Oy lấy hai điểm N vµ P cho ON = cm, OP = cm

a) Chøng tá r»ng ®iĨm N trung điểm đoạn thẳng MP

b) Trờn tia đối tia My lấy điểm Q cho MQ = cm Tìm trung điểm đoạn thẳng PQ, MN, NQ? Giải thích sao?

2) Cho hai tia Ox Oy đối Trên nửa mặt phẳng bờ đ−ờng thẳng xy, vẽ tia OA, OB, OC cho xOA 45 ; xOB 80 ; yOC 65 = o = o = o

a) Tia OA có tia phân giác góc xOB không? Vì sao? b) Chứng tỏ tia OB tia phân giác góc AOC

c) Vẽ tiếp 2016 tia gốc O khơng có hai tia trùng kể tia cho Hỏi có tất góc c to thnh?

Bài 6.(1,0 điểm)

(26)

đề thi khảo sát chọn học sinh giỏi lớp 7

huyện xuân tr−ờng, tỉnh nam định

Năm hc 2018 - 2019

Môn thi: Toán

Thời gian làm bài: 120 phút

Bài 1.(6,0 điểm)

1) Tính giá trị biểu thức

3

0,375 0,3 1,5 0,75 11 12

A

5 5 0,625 0,5 2,5 1,25

11 12

− + + + −

= +

− + − − + −

2) T×m x, biÕt: x 5

− −

− − =

3) Tìm số nguyên x, biết: 49 x 26 < < 81

Bài 2.(3,0 điểm)

Cho x y hai đại l−ợng tỉ lệ nghịch Gọi x1 x2 hai giá trị t−ơng ứng x; y1 y2

hai giá trị tơng ứng y

a) Tính x1 vµ y1, biÕt 2x1= 5y1 vµ 2x1− 3y1= 12

b) TÝnh y1, biÕt x1= 2x2 vµ y2= 10 Bµi 3.(4,0 ®iĨm)

Cho tam giác ABC vng A có AB < AC Kẻ AH vng góc với BC (H ∈ BC) Lấy điểm D AC cho AD = AB Kẻ DE DK lần l−ợt vuông góc với BC AH (E ∈ BC, K ∈AH) a) So sánh độ dài BH AK

b) Tính số đo góc HAE

Bài 4.(4,0 ®iĨm)

Cho tam giác ABC có B 45 ; C 15 = o = o Trên tia đối tia AB lấy hai điểm M, D cho BA = AM = MD Kẻ DE vng góc với AC E

a) Chứng minh tam giác AME b) Chứng minh EC = ED

Bài 5.(3,0 điểm)

(27)

thi OLympic lớp

quận hoàng mai, TP hà nội

Năm học 2018 - 2019 Môn thi: Toán

Thời gian làm bài: 150 phút Bài 1.(4,0 điểm)

1) Cho a, b, c tháa m·n a − b = b c = Tính giá trị cđa biĨu thøc = + + − − −

− − +

2 2

2

a b c ab bc ca

T

a c 2ab 2bc

2) Cho S = 23n + 1 + 23n − + víi n số nguyên dơng Chứng minh S

Bài (5,0 điểm)

1) Giải phơng tr×nh (12x + 7)2 (3x + 2)(2x + 1) =

2) Tìm số nguyên dơng x, y tháa m·n x2y + 4xy + 4y = 162x + 162

Bài 3.(3,0 điểm)

Cho x, y số dơng thỏa mÃn x3+ y3+ 6xy =

1) Chøng minh x + y =

2) Tìm giá trị nhỏ biÓu thøc = + + 2

1

A

xy

x y

Bµi 4.(6,0 điểm)

Cho tam giác ABC nhọn (AB < AC) có H trực tâm Qua A kẻ đờng thẳng song song với BH CH tạo với đờng thẳng hình bình hành AEHF (AE // BH)

1) Chøng minh ΔEHA ΔABC

2) Kẻ trung tuyến AM tam giác ABC Chứng minh AM EF 3) Kẻ HI vuông góc víi AM t¹i I Chøng minh MC2 = MI.MA

Bài 5.(2,0 điểm)

Mt gii búng chuyn cú đội bóng tham gia thi đấu vịng trịn l−ợt, trận đấu có kết thắng thua (hai đội thi đấu với trận) Biết đội thứ thắng a1 trận thua b1 trận, đội thứ hai thắng a2 trận thua b2 trận, , đội thứ chín thắng a9

(28)

thử sức tr

ớc kì thi vào 10 THPT chuyên

Năm học 2020 - 2021

Môn thi: Toán

Thời gian làm bài: 150 phút

Nguyễn Đức Tấn

Bài (2 điểm)

1) Cho a x= + 1, b y= + 1, c xy= +

x y xy Tínhgiátrị cđa biĨu thøc M = a

2+ b2+ c2− abc

2) Giải phơng trình nghiệm nguyên (2x + y)3+ 2019y3= x3y3 Bài (2 điểm)

1) Giải hệ phơng trình =

− + = −

⎪⎩

2

(x 2y)(y 4y) x y 2y

2) Cho ph−ơng trình x2+ (2m − 1)x −5m − = (m tham số) Tìm m để ph−ơng trình có hai nghiệm x1, x2 thỏa mãn x1 − x2 = x12

Bµi (2 điểm)

1) Cho a, b hai số hữu tØ kh¸c tháa m·n a3+ b3= 2a2b2 Chøng minh 1

ab số hữu tỉ

2) Cho tam giác ABC nội tiếp đờng tròn (O) Tia phân giác góc A cắt đờng tròn (O) D Chứng minh AB + AC < 2AD

Bài (2 điểm)

Cho điểm A nằm ngồi đ−ờng trịn (O) Vẽ hai tiếp tuyến AB, AC với đ−ờng tròn (O) (B, C tiếp điểm) AO cắt đ−ờng tròn (O) D, E (D nằm A E) Một đ−ờng thẳng thay đổi qua A cắt đ−ờng tròn (O) M, N (M thuộc cung BD) Chứng minh tâm I đ−ờng tròn ngoại tiếp tam giác OMN thuộc đ−ờng cố định đ−ờng thẳng BC, DN, ME đồng quy Bài (2 điểm)

1) Cho a, b, c tháa m·n a2 + b2+ c2 = Tìm giá trị lớn giá trị nhỏ nhÊt cđa biĨu thøc M = a + b + c − abc

2) Trong hình vng có cạnh 1, đặt 2019 đ−ờng tròn, đ−ờng tròn có đ−ờng kính

(29)

HƯớNG DẫN GIảI đề thi

thư søc tr−íc k× thi vào 10 THPT chuyên

Năm học 2020 - 2021

Môn thi: Toán (Đề đăng TTT2 số 203)

Bµi 1) + + =

+ + + + +

1 1

a b b c c a a b c

⇒ + + = + + + ⎛ ⎞ ⇒ + + ⎜ + + ⎟= + + + + + ⎝ ⎠ ⇒ + + = + + +

2 2

c a b

1

a b b c c a

c a b

(a b c) a b c

a b b c c a

a b c

0

b c c a a b

Do = ⎛⎜⎜ + + ⎞⎟⎟

+ + +

⎝ ⎠

2 2

a b c

M

b c c a a b + 2(b − c

+ c − a + a − b) =

2) XÐt p = ta cã p2 + 59 = 63 cã ớc

dơng 1; 3; 7; 9; 21; 63

XÐt p = ta cã p2+ 59 = 68 có ớc dơng 1; 2; 4; 17; 34; 68

Xét p > Ta có p2 số ph−ơng lẻ nên p2 chia d− chia d− 1, từ p2+ 59

chia hÕt cho vµ hay p2+ 59 chia hÕt cho

12 Suy p2+ 59 cã nhiỊu h¬n −íc dơng

Vậy p = p =

Bµi 1) ⎛⎜ + ⎞⎟ + =

⎝ ⎠

3

1

1 (1 x ) 16

x ⎛ ⎞ ⎛ ⎞ ⇔⎜ + ⎟ + ⎜ + ⎟ − = ⇔ = ⎝ ⎠ ⎝ ⎠ 1

x x 20 x

x x

2) (x2+ 4x −12)(x2+ 12x + 20) = m (1)

Phơng trình (1) tơng đơng với

(x − 2)(x + 6)(x + 2)(x + 10) = m ⇔ (x2+ 8x +12)(x2+ 8x − 20) = m Đặt t = x2+ 8x +16 = (x + 4)20

Phơng trình trở thành (t 4)(t − 36) = m ⇔t2− 40t + 144 − m = (2)

(1) cã nghiƯm ph©n biệt (2) có hai

nghiệm dơng phân biệt

256 m

S 40 256 m 144

P 144 m Δ = + >

⎧ ⎪

⇔⎨ = > ⇔ − < <

⎪ = − > ⎩

Bµi 1) 3(ab + bc + ca)2≤ (a + b + c)2≤

Do ≤

+ + + +

2

bc bc

a a ab bc ca

⎛ ⎞

= ≤ ⎜ + ⎟

+ +

+ + ⎝ ⎠

bc bc 1

2 a b a c

(a b)(a c)

T−¬ng tù ≤ ⎛⎜ + ⎞⎟

+ +

⎝ ⎠

+

2

ca ca 1

;

2 b c b a

b ≤ ⎛⎜ + ⎞⎟ + + ⎝ ⎠ +

ab ab 1

2 c b c a

c

Cộng vế với vế bất đẳng thc trờn ta

đợc M a b c

2 2

≤ + + ≤ Đẳng thức xảy

khi a = b = c =

(30)

2) Ta cã + + +

3

1 7b 3c

100bc a (7b 3c)

+

≥ =

+

3

1 7b 3c

2

100bc 5a

a (7b 3c)

⇒ ≥ − −

+

3

1

5a 100c 100b

a (7b 3c)

T−¬ng tù ta cã

≥ − −

+

3

1

; 5b 100a 100c b (7c 3a)

≥ − −

+

3

1

5c 100b 100a c (7a 3b)

Cộng vế với vế bất đẳng thức ta đ−ợc

+ +

+ + +

⎛ ⎞

≥ ⎜ + + ⎟

⎝ ⎠

3 3

1 1

a (7b 3c) b (7c 3a) c (7a 3b)

1 1

10 a b c

Đẳng thức xảy a = b = c =

Bài Dễ chứng minh đ−ợc OA vng góc với EF nên OA trung trực PQ từ suy tam giác APQ cân A

Tø gi¸c ACDF néi tiÕp

⇒BFD ACB AQM Suy tø gi¸c AQMF = =

nội tiếp nên AMQ AFQ.=

Tứ giác BCEF néi tiÕp nªn AFQ ACB =

Từ suy

= = = ⇒

AMQ AFQ ACB AQM AMQ cân

A hay AM = AQ

Tơng tự ANP cân A hay AN = AP

Kết hợp với APQ cân t¹i A ta cã AM = AN =

AP = AQ

VËy M, N, P, Q cïng thuéc đờng tròn tâm A

Bài 1) Đặt n = 3k + r (k ∈N; r ∈ {0; 1; 2}) Ta cã 5n+ = 53k + r+

= 5r(125k− 1) + 5r+

Mµ 125k − chia hÕt cho 125 − =124 nªn

125k− chia hÕt cho 31

Lại có r {0; 1; 2} nên 5r+ ∈ {5; 9; 29}

VËy 5n+ chia cho 31 d− hc hc 29

2) Ta cã 673 + 673 + 673 = 2019;

367 + 367 + 367 = 1101

Do ta tô màu đỉnh đa giác hai màu đỏ xanh: Tô màu đỏ đỉnh ghi số 673, tơ màu xanh đỉnh ghi số 367 Bài tốn giải xong chứng minh đ−ợc tồn ba đỉnh đa giác ba đỉnh tam giác cân, ba đỉnh tơ màu

Thật vậy: Vì đa giác có 2019 đỉnh số lẻ

nên tồn hai đỉnh kề đ−ợc tô

màu, gọi hai đỉnh A B Đa giác

cịn có đỉnh C nằm đ−ờng trung trực

AB

Nếu C màu với A B tam giác ABC tam giác cân có ba đỉnh tơ màu

Nếu C khác màu với A B, ta xét đỉnh D kề với đỉnh A (D khác B), đỉnh E kề với đỉnh B (E khác A) Nếu D, E khác màu với A B tam giác DCE tam giác cân có ba đỉnh tơ màu, có hai đỉnh D E màu với A B, chẳng hạn D tam giác ADB cân có ba đỉnh màu

Nh− tồn ba đỉnh đa giác ba đỉnh

của tam giác cân, ba đỉnh đ−ợc tô

(31)

Sử dụng tính chất hàm số

bậc để giải tốn

Thái nhật ph−ợng (Khánh Hịa) ính chất hàm số bậc nhất, đặc biệt

là tính chất đồng biến hay nghich biến đ−ợc sử dụng để giải nhiều toán hiệu Chúng ta nhắc lại tính chất quan trọng

TÝnh chÊt cđa hµm sè bËc nhÊt

Hàm số bậc y = ax + b (a ≠ 0) đồng biến R a > nghịch biến trờn R nu a <

Hệ quả:Xét hàm sè y f(x) ax b = = + (a 0) , ≠

[ ]

∀ ∈ α βx ,

NÕu f( ) vµ α ≥ f( ) th× β ≥ f(x) ≥

NÕu f( ) vµ α ≤ f( ) th× β ≤ f(x) ≤

Nếu hàm số y f(x) đồng biến =

nghịch biến tập xác định D mà

α = β

f( ) f( ) víi α,β D = Một số toán ¸p dơng

Bµi to¸n Cho hµm sè bËc nhÊt y = f(x) =

(6m − m2− 10)x + m (m lµ h»ng sè) H·y so

sánh f(A) f(B) với

= + +

A 2016 2018 2020;

= + +

B 2013 2015 2026

Lêi gi¶i.Ta cã 2016− 2013

= >

+ +

3

;

2016 2013 2020 2026

2018 2015

= >

+ +

3

2018 2015 2020 2026

Do

− + −

> = −

+ ⇒ >

2016 2013 2018 2015

6

2026 2020

2020 2026

A B

Hµm sè bËc nhÊt y = f(x) = (6m − m2−10)x +

m cã hÖ sè a = 6m − m2− 10 =−(m − 3)2− <

nên f(x) nghịch biến tập R Từ suy f(A) < f(B)

Bài toán Chøng minh víi m ≤ −3 vµ

− ≤ <3 x th× x

2− 2(m + 2)x + m

Lời giải.Vì ≤ <3 x

2 nªn −2x + > Suy hµm sè bËc nhÊt f(m) = (−2x + 1)m +

x2− 4x đồng biến

Do f(m) ≤ f(−3) = (−2x + 1)(−3) + x2− 4x

= (x − 1)(x + 3) ≤

hay x2− 2(m + 2)x + m (đpcm)

Bài toán Cho x, y, z số thực không

©m tháa m·n x + y + z = Chøng minh r»ng

+ + + ≥

3 3 15

x y z xyz

4

Lời giải.Bất đẳng thức cho t−ơng đ−ơng

víi (x y)+ −3xy(x y) z+ + 3+15xyz≥

4

⇔ −(1 z)3−3xy(1 z) z− + 3+15xyz≥1

4

(32)

⎛ ⎞

⇔⎜ − ⎟ + − + ≥

⎝ ⎠

2

9

z xy z z

4

Đặt = + = ⎞⎟

⎝ ⎠ ⎝ ⎠

2

x y z

t xy t

2

XÐt =⎛⎜ − ⎞⎟ + − +

⎝ ⎠

2

9

f(t) z t z z

4

NÕu z=

9 th×

⎛ ⎞

=⎜ ⎟ − + = > ⎝ ⎠

2

4 1

f(t)

9 324

NÕu z≠

9 f(t) hàm số bậc có

⎛ ⎞

=⎜ − ⎟ ≥

⎝ ⎠

2

1

f(0) z

2 vµ

⎡⎛ − ⎞ ⎤ ⎛ ⎞⎛ − ⎞ ⎢⎜ ⎟ ⎥=⎜ − ⎟⎜ ⎟ + − + ⎢⎝ ⎠ ⎥ ⎝ ⎠⎝ ⎠ ⎣ ⎦ 2

1 z z

f z z z

2 4

= z (3z 1)− 2≥0

16 (Do z ≥ 0)

Do f(t) (đpcm) ≥

Bµi to¸n Cho ≤ x, y, z, t ≤ Chøng minh r»ng

x + y + z + t + (1 − x)(1 − y)(1 − z) (1 t)

Lời giải Xét hàm sè f(x) = [1 − (1 − y)(1 −

z)(1 − t)]x + y + z + t + (1 − y)(1 − z)(1 − t) NÕu (1 y)(1 z)(1 t) th× − − − − =

= + + + ≥

f(x) y z t 1

NÕu − (1 − y)(1 − z)(1 − t) ≠ th× f(x) lµ hµm sè bËc nhÊt cã f(0) = y + z + t + (1 − y)(1

− z)(1 − t) = + yz + yt + zt(1 − y) ≥ vµ f(1) y z t = + + + ≥

Do ú f(x) (pcm)

Bài toán Tìm c¸c sè x, y, z ∈ [0; 1] tháa m·n x y z (xy yz zx) + + + + =

Lời giải Xét hàm số

= − − + + − −

f(x) (1 y z)x y z yz NÕu y z th× − − = f(x)= −yz ≤

NÕu y z th× f(x) lµ hµm sè bËc − − ≠

nhÊt cã f(1) =−yz ≤ 0;

f(0) = y + z − yz − =−(1 − y)(1 − z) ≤ Do f(x) ≤

Đẳng thức xảy ba số x, y, z cã mét sè b»ng 0, mét sè b»ng vµ mét sè tïy ý thuéc

[ ]

0;1

Vậy (x, y, z) = (0; 1; m) hoán vị với m

[0; 1]

Bài toán Cho số thực x, y, z không

âm thỏa mÃn x + y + z = Tìm giá trị nhỏ A = x2+ y2+ z2+ xyz

Lêi gi¶i Ta cã

− = + 2− + 2+ −

A (x y) 2xy z xyz

= − − + + −

= − + − +

2

2

(3 z) 2xy z xyz

(z 2)xy 2z 6z

Đặt t xy, suy =

+ −

⎛ ⎞ ⎛ ⎞

≤ ≤⎜ ⎟ =⎜ ⎟

⎝ ⎠ ⎝ ⎠

2

x y z

0 t

2

XÐt hµm sè f(t) (z 2)t 2z= − + 2−6z +

NÕu z th× = f(t) = >

NÕu z f(t) hàm số bậc có

⎛ ⎞

= − + = ⎜ − ⎟ + >

⎝ ⎠

2

2

f(0) 2z 6z z 0;

2 ⎡⎛ − ⎞ ⎤ ⎛ − ⎞ ⎢⎜ ⎟ ⎥= − ⎜ ⎟ + − + ⎢⎝ ⎠ ⎥ ⎝ ⎠ ⎣ ⎦ 2

3 z z

f (z 2) 2z 6z

2

= 1(z 1) (z 2) 0− + ≥

4 (Do z ≥ 0)

Do f(t) 0≥ ⇒A ≥

Đẳng thức xảy x = y = z = VËy Amin= x = y = z =

Bµi toán 7.Giải phơng trình

+ = +

− + − +

2

5x x 6x 2x

2 2x 10 2x

Lời giải ĐKXĐ x −1

(33)

Ph−ơng trình cho t−ơng đ−ơng với

− − = + − + −

5x (x 3) 2x ( 2x 3)

⇔5x | x | 2x | 2x | − − = + − + −

XÐt hµm sè

+ ≥

= − − =⎨ − < ⎩

3t (t 3) f(t) 5t t

7t (t 3)

Suy f(t) hàm đồng biến ph−ơng trình có dạng f(x) f( 2x 1) = +

Do = + ⇔⎧⎪⎨ ≥

− − = ⎪⎩

x

x 2x

x 2x

⇔ = +x

Vậy phơng trình có nghiệm x 1= +

Bài toán 8.Giải phơng trình

+

= + +

− + 2

x 2x

(x 1)( x 2)

x 2x

Lời giải ĐKXĐ x

Phng trỡnh ó cho t−ơng đ−ơng với

− + = + − + + − + = ⎡ ⎢ ⇔⎢ + = + ⎢ − + + + ⎣ 2

(x 2)(x 4) (x 1)(x 2)

x 2

x 2x

x

x x

(1) x 2

x 2x

Tõ (1) ta cã

+ + + = + − + ⇔ + + + + = − + − + 2

(x 4)( x 2) (x 1)(x 2x 3)

( x 2)( (x 2) 2)

(x 2)[(x 1) 2] (2)

XÐt hµm sè f(t) = (t + 2)(t2+ 2) = t3+ 2t2+ 2t +

Víi t1< t2 ta cã

− = − + − + −

= − + + + + +

3 2

1 2 2

2

1 2 2

f(t ) f(t ) (t t ) 2(t t ) 2(t t )

(t t )(t t t t t 2t 2)

⎡ ⎤

=12(t t ) t1− ⎣12+ +t22 (t12+2t t1 2+t ) 4(t t ) 422 + 1+ + ⎦

⎡ ⎤

= 12(t1−t ) t2 ⎣12+t22+ + +(t1 t2 2)2⎦<0

⇒f(t ) f(t )1 < 2 nên f(t) hàm số đồng biến ph−ơng trình (2) có dạng f( x 2) f(x 1) + = −

Do ≥ ⎧ + ⎪ + = − ⇔⎨ ⇔ = − − = ⎪⎩

x 3 13

x x x

2

x 3x

Đối chiếu với điều kiện ta đ−ợc nghiệm ph−ơng trình cho x = 2; x= 3+ 13

2

Bµi tËp tù lun

Bµi Cho y f(x) (m= = 2−3m 9)x m So + −

s¸nh ⎛⎜ + + ⎞⎟

⎝ ⎠

5

f

6 vµ f( 5+ 6+ 7)

Bµi Cho m ≤ 1; x ≥1 Chøng minh r»ng

− − + + ≥

2

x 2(3m 1)x m

Bài Cho số thực x, y, z không âm thỏa

mÃn x + y + z = Chøng minh r»ng: a) 5(x2+y2+z ) 6(x2 ≤ 3+y3+z ) +

b) 7(xy yz zx) 9xyz + + ≤ +

c) x y y z z x2 + + ≤ 27 d) x2+y2+z2+xyz≥ 10

27

Bài Tìm x, y, z [0; 2] tháa m·n

2(x + y + z) − (xy + yz + zx) =

Bµi Cho x, y, z [0; 1] Tìm giá trị lín nhÊt

cđa biĨu thøc P = x2(1 − y) + y2(1 − z)

+ z2(1 x)

Bài Giải phơng tr×nh

− − = − +

− − − −

2

3(x 5x 4) x 2x

5x 5x

(34)

Bµi 1(202) Cho A = 5n2+ 10n + 601, với n

số tự nhiên Tìm giá trị nhỏ tổng chữ số A

Lời giải Vì n số tự nhiên nên A chia d− Do A ≠10k (với k ∈N*) A > nên tổng chữ số A lớn

Víi n = th× A = 5.82+ 10.8 + 601 = 1001 Vậy giá trị nhỏ tổng chữ số số A

Nhận xét Các bạn sau có lời giải tốt: Lê Văn Quang Hiếu, 7D, THCS Lý Nhật Quang, Đô

Lơng, Nghệ An; Đỗ Ngọc Tiến, 7A3, THCS

Lâm Thao, Lâm Thao, Phó Thä; D−¬ng

Trung Qc, 6D, THCS LËp Thạch, Lập Thạch; Nguyễn Huy Hoàng Sơn, 6A2, THCS

Yên Lạc, Yên Lạc, Vĩnh Phúc; Lê Phú

Quang, 7D; Trần Minh Hoàng, 7E, THCS

Nguyễn TrÃi, Nghi Xuân, Hà Tĩnh; Nguyễn

Hoàng Nhật Nam, 6A12, THCS Vinschool

Times City, Q Hai Bà Trng; Nguyễn Trần

Kiên; Hoàng Anh Khôi, 7C1, THCS

Archimedes Academy, Q Cầu Giấy, Hà Nội phùng kim dung Bài 2(202) Tìm số nguyên x, y, z thỏa mÃn x2013 +y2016+z2019 =20182021

Lêi gi¶i (Theo lêi gi¶i cđa bạn Trần Minh Hoàng, 7E, THCS Nguyễn TrÃi, Nghi Xuân, Hµ TÜnh)

Ta cã 2018 (mod 9) ≡

2018202122021 (mod 9)

Mặt khác

= = =

⇒ ≡ − ≡ ≡

2021 2016 224 224

2021 224

2 2 (2 ) 512

2 ( 1) 1.32 (mod 9)

Suy 20182021≡5(mod 9) (1)

Víi a số nguyên a có dạng 9k; 9k ± 1; 9k ± 2; 9k ± 3; 9k 4, với k số nguyên

Do a3 ≡ m3 (mod 9), với m ∈ {0; ±1; ±2;

±3; ±4}

Suy a3 chia cho chØ cã thÓ d− 0; 1;

Từ suy với số nguyên x, y, z x2013 + y2016 + z2019 = (x671)3 + (x672)3 + (x673)3

chia cho chØ cã thÓ d− 0; 1; 2; 3; 6; 7; (2)

(35)

Nhận xét Đây toán số học hay, số bạn biến đổi khơng xác nên

khơng số d− chia 20182021 cho 9,

ngoài bạn Hoàng, bạn sau có lời giải tốt: Lê Văn Quang Hiếu, 7D, THCS Lý Nhật

Quang, Đô Lơng, Nghệ An; Nguyễn Huy

Hoàng Sơn, 6A2, THCS Yên Lạc, Yên Lạc, Vĩnh Phúc; Nguyễn Trần Kiên, Nguyễn

Quang Minh, Hoàng Anh Khôi, 7C1, THCS

Archimedes Academy, Q Cầu Giấy, Hà Nội nguyễn ngọc hân Bài 3(202) Viết ngÉu nhiªn chÝn sè 2, 5, 8, 11, 14, 17, 20, 23, 26 vào chín ô vuông hình bên, ô chứa số Gọi x tổng số hàng, y tổng số cột, z tổng số đờng chéo Đặt S = x + y + z Tìm giá trị lớn S

Li gii Ta thấy tổng số hàng tổng số cột tổng số cho

Suy x = y = + + + 11 + 14 + 17 + 20 + 23 + 26 = 126

Do S = x + y + z = 252 + z Vậy để S lớn z phải lớn

Khi tÝnh tỉng c¸c số hai đờng chéo số

ụ đ−ợc tính lần nên để z lớn

nhất số phải số lớn số số ú l 26

Các số lại nằm hai đờng chéo phải

là 14, 17, 20, 23

Do zmax = 14 + 17 + 20 + 23 + 26 + 26

= 126

VËy Smax = 252 + 126 = 378

Hình dới minh họa cách điền số

vào ô vuông để Smax= 378

Nhận xét Đa số bạn tham gia giải Một số bạn tính z tính số lần nên dẫn đến kết sai

Các bạn sau có lời giải đúng: Đào Quang

Minh, 6A; D−¬ng Trung Quèc, 6D, THCS

Lập Thạch, Lập Thạch; Bạch Thái Sơn, 7A1, THCS Vĩnh Yên, TP Vĩnh Yên, Vĩnh Phúc; Đỗ Ngọc Tiến, 7A3, THCS Lâm Thao, Lâm

Thao, Phú Thọ; Đoàn Nguyªn Vị, 6B,

THCS Ngun HiỊn, Nam Trùc, Nam Định;

Lê Văn Quang Hiếu, Nguyễn Cảnh Nam

Khánh, 7D; Lê Khắc Hng, 7A, THCS Lý

Nhật Quang, Đô Lơng; Nguyễn Ngọc Việt,

7B, THCS Đặng Thai Mai, TP Vinh, Nghệ An; Đoàn Minh Đức, 7A, THCS Hoàng Xuân

HÃn, H Đức Thọ, Hà Tĩnh; Nguyễn Trần

Kiên; Nguyễn Quang Minh; Hoàng Anh Khôi,

7C1; Phạm Đăng Việt Bách, 6C4, THCS

(36)

Bài 4(202).Cho tam giác ABC có số đo độ dài cạnh số tự nhiên Gọi D điểm tùy ý cạnh BC (D không trùng với B C) Trên tia đối tia DA lấy điểm M cho

= o

AMB 60 Chøng minh r»ng MA2 + MB2 +

MC2 không số phơng

Lời gi¶i

Đặt cạnh tam giác a (a *)

Trên cạnh AM lấy điểm N cho MN = MB

khi tam giác MNB đều, suy MB = NB

KÕt hỵp víi CB = AB, CBM ABN =

= o−

( 60 NBC) nªn ta cã

ΔCBM = ΔABN (c.g.c) Từ AN = CM

Suy AM = AN + NM = MB + MC KỴ BH ⊥ MN (H ∈ MN)

Vì tam giác MNB nên ta có

= = =

HM HN MN MB

2

Đặt MB = x HM= x,

2 MC = y th× MA = x + y

vµ AH= +x y

2

áp dụng định lí Pythagoras cho tam giác HMB HAB vng H ta có:

= − = − =

⎛ ⎞

= + =⎜ + ⎟ +

⎝ ⎠

2

2 2

2

2 2

x 3x

BH MB MH x ;

4

x 3x

AB AH BH y

2

⇔x2+xy y+ 2=a Từ suy

+ + = + + +

= + + =

2 2 2

2 2

MA MB MC (x y) x y

2(x xy y ) 2a

Do a N* nên MA2+ MB2+ MC2 không số

chính phơng

Nhận xét Điểm mấu chốt toán MA = MB + MC Các bạn sau có lời giải

ỳng: Trn Thị Thanh Thảo, 8A3, THCS

L©m Thao, L©m Thao, Phó Thä

hå quang vinh

Bµi 5(202) Giải phơng trình

+ + =

1

1 2x 2

x x

Lời giải ĐKXĐ: x<

áp dụng bất đẳng thức AM-GM cho hai s

dơng x x ta có

= + − ≥ − ⇒ ≥

1 x (1 x) x(1 x)

x(1 x)

Đẳng thức xảy chØ x=

2

áp dụng bất đẳng thức AM-GM cho hai số

d−¬ng

x vµ −

1

1 x ta cã

+ ≥ ≥

− −

1 1

2 2

x x x(1 x)

Đẳng thức xảy x=

(37)

Suy + + − ≥ −

1

1 2x 2

x x

Do + + − = ⇔ =

1 1

1 2x 2 x

2

x x

Vậy tập nghiệm phơng trình lµ

⎧ ⎫

=⎨ ⎬

⎩ ⎭

S

2

Nhận xét Đây toán quen thuộc, hầu hết bạn học sinh có lời giải

đúng theo h−ớng đánh giá Có nhiều bạn

qn khơng đủ thụng tin trng hoc huyn

tỉnh, TP nên không đợc khen kì Các

bn sau cú li giải đúng, trình bày tốt: Phạm Duy Nguyên Lâm, 8A1, THCS Thanh Xuõn,

Q Thanh Xuân; Nguyễn Thái Hoàng, 9A5,

THCS Cầu Giấy; Nguyễn Trần Kiên, 7C1,

THCS Archimedes Academy, Q Cầu Giấy, Hà Nội; Nguyễn Ngọc Diệu, 9A, THCS Nguyễn Hiền, Nam Trực, Nam Định; Nguyễn Đức An, 9A, THCS Lê Văn Thịnh, Gia Bình, Bắc Ninh; Nguyễn Quốc Bảo Long, 9C, THCS Đặng Thai Mai, TP Vinh; Phạm Ngọc

Trinh, 9B, THCS Hồ Xuân H−¬ng, Qnh

L−u, NghƯ An; Kim Anh Hïng, 8A1, THCS Vĩnh Yên, TP Vĩnh Yên; Tạ Kim Nam Tuấn,

7A2, THCS Yên Lạc, Yên Lạc, Vĩnh Phúc;

Vũ Đình Hoàng, Nguyễn Mạnh Hùng, 9A3,

THCS Giấy Phong Châu, Phù Ninh; Phùng

Đăng Dơng, 8C, THCS Văn Lang, TP Việt

Trì, Phú Thọ

Trịnh Hoài Dơng

Bài 6(202) Cho số thực x, y, z, t tháa m·n < x, y, z, t Tìm giá trị nhỏ biÓu thøc = +

+ + +

xyzt

Q

x y z t

Lời giải Tõ gi¶ thiÕt, ta cã

(1 − x)(1 − y) ≥ ⇔ xy + ≥ x + y (1 − xy)(1 − z) ≥ ⇔ xyz + ≥ xy + z (1 − xyz)(1 − t) ≥ ⇔ xyzt + ≥ xyz + t Cộng vế với vế bất đẳng thức ta đ−ợc xyzt + ≥ x + y + z + t hay

+

= ≥

+ + + xyzt

Q

x y z t

Đẳng thức xảy vµ chØ x = y = z = t = Vậy giá trị nhỏ Q

Nhận xét Bằng cách giải tơng tự, ta giải đợc toán tổng quát: Nếu x1, x2, , xn

là số thực dơng không lớn

n + x1x2 xn≥ x1+ x2+ + xn

Các bạn sau có lời giải tốt: Nguyễn Tuấn Đạt, 9B, THCS Nguyễn Quang Bích, Tam

Nông; Phùng Đăng Dơng, 9C, THCS Văn

Lang, TP Việt Trì, Phú Thọ

(38)

Bài 7(202) Giải hệ phơng trình

+ + =

− + − + − =

⎪⎩

2 2

3 3 3

x y z

| x y | | y z | | z x | 32

Lời giải Do vai trß cđa x, y, z nh nhau,

không tính tổng quát ta giả sư x ≥ y ≥ z

Khi x3 ≥ y3 ≥ z3 Từ ta có

= − + − + −

= − + − + − = −

3 3 3

3 3 3 3

32 | x y | | y z | | z x |

(x y ) (y z ) (x z ) 2(x z )

⇔ − = =

= + + ≥ +

⇔ − ≥ +

⇔ ≥ + − −

= + + + − + −

⎡ ⎤

= ⎣ + + + ⎦

⇒ = ⇒ =

3 3

2 2 2

3 2

2 3

6 2 6 3

2 2 2

x z 16 (2 2)

( x y z ) ( x z )

(x z ) (x z )

0 (x z ) (x z )

x 3x z 3x z z x 2x z z

x z 2x 2z (x z)

xz y

• Víi x = 0, y = z= 2

ã Với z = 0, y = th× x 2 =

Vậy hệ ph−ơng trình cho có nghim l

(2 2, 0, 0) hoán vị; ( 2, 0, 0)

các hoán vị

Nhn xột õy l bi toỏn giá trị tuyệt đối dạng Các bạn sau có lời giải tốt: Ngơ Thị An Bình, 8E, THCS Đặng Thai Mai, TP

Vinh, NghƯ An; Ph¹m Duy Nguyên Lâm,

8A1, THCS Thanh Xuân, Q Thanh Xuân; Lê Duy Anh, 9C1, THCS Archimedes Academy, Q Cầu Giấy, Hà Nội

Nguyễn Minh Đức

Bài 8(202).Cho tam giác nhọn ABC (AB < AC) nội tiếp đờng tròn (O) Các đờng cao BD, CE ABC cắt H Gọi M trung điểm BC, N giao điểm AH DE, K giao ®iĨm cđa AO vµ BC Chøng minh r»ng HM // NK

Lời giải Dựng đờng kính AF đờng tròn (O) Ta dễ thấy BH CF vuông gãc víi AC nªn BH // CF

T−ơng tự CH // BF, tứ giác HBFC l

một hình bình hành

Vậy M trung điểm chung BC HF

Ta cã AFC ABC= (cïng ch¾n cung AC),

=

ABC AHE (cïng phơ víi gãc BAH) nªn =

AFC AHE mµ AEH ACF 90 = = o

Do ΔAEH ΔACF (g.g) ⇒ AE= AC (1)

AH AF Lại có tứ giác BCDE nội tiÕp nªn

=

AEN ACK dẫn đến ΔAEN ΔACK (g.g)

Do AN AK (2)

AE= AC

Tõ (1) vµ (2), suy AN= AK

AH AF

(39)

NhËn xÐt Các bạn sau có lời giải tốt: Nguyễn Duy Khôi, 8G, THCS Đặng Thai

Mai, TP Vinh, Nghệ An; L¹i Tr−êng Giang,

9A, THCS Ngun Quang BÝch, Tam Nông, Phú Thọ; Lê Thành Trung, 9D, THCS Điện

Biên, TP Thanh Hóa, Thanh Hóa; Phạm

Duy Nguyên Lâm, 8A1, THCS Thanh Xuân,

Q Thanh Xuân; Lê Duy Anh, Ngun L−¬ng

Uy, 9C1, THCS Archimedes Academy, Q Cầu Giấy, Hà Nội

trần quang hùng

Cách đặt mua tạp chí năm 2020

Đặt b−u in ca VNPT: Tp

Toán Tuổi thơ 1: C169; Tạp chí Toán

Tuổi thơ 2: C169.1; Tổng tập Toán Tuổi

thơ năm 2018: C169.2; Tổng tập Toán

Tuổi thơ năm 2018: C169.3; Tổng tập

Toán Tuổi thơ năm 2017: C169.4; Tổng tập Toán Tuổi thơ năm 2017: C169.5

Các tập thể nhμ tr−ờng đặt qua Phịng GD vμ ĐT Sở GD vμ ĐT Thông tin chi tiết liên hệ qua số máy 0912268725 (gặp chị Trnh Th Tuyt Trang)

Đ

ợc th

ởng kì

T

hi giải toán qua th

Hoàng Anh Khôi, Nguyễn Trần Kiên, 7C1, THCS Archimedes Academy, Q Cầu

Giấy; Phạm Duy Nguyên Lâm, 8A1,

THCS Thanh Xuân, Q Thanh Xuân, Hà

Nội; Lê Văn Quang Hiếu, 7D, THCS Lý

Nhật Quang, Đô Lơng, Nghệ An; Đỗ

Ngọc Tiến, 7A3, THCS Lâm Thao, Lâm

Thao; Phùng Đăng Dơng, 8C, THCS

Văn Lang, TP Việt Trì, Phú Thọ; Dơng

Trung Quốc, 6D, THCS Lập Thạch, Lập

Thạch; Nguyễn Huy Hoàng Sơn, 6A2,

THCS Yên Lạc, Yên Lạc, Vĩnh Phúc;

Nguyễn Đức An, 9A, THCS Lê Văn Thịnh, Gia Bình, Bắc Ninh; Đoàn Minh Đức, 7A,

THCS Hoàng Xuân HÃn, Đức Thọ, Hà

Tĩnh; Đoàn Nguyên Vũ, 6B, Ngun Ngäc DiƯu, 9A, THCS Ngun HiỊn, Nam Trùc, Nam Định; Lê Thành Trung, 9D, THCS

Điện Biên, TP Thanh Hãa; Lª Phó

(40)

chứng minh đ

ờng thẳng qua Điểm cố định

NGUYễN XUÂN CảNH

Trờng THCS Lý Tự Trọng, Bình Xuyên, Vĩnh Phúc

rong tốn hình học dạng tốn chứng minh đ−ờng thẳng qua điểm cố định dạng khó, hay gặp đề thi học sinh giỏi cấp tỉnh thi vào tr−ờng THPT chuyên

Để chứng minh đ−ờng thẳng qua điểm cố định ta th−ờng theo b−ớc sau:

- Tìm hiểu nội dung tốn: Trong b−ớc cần xác định rõ yếu tố cố định, quan hệ yếu tố không đổi với yếu tố thay đổi, tìm mối quan hệ yếu tố

- Dự đốn điểm cố định: Dựa vào vị trí đặc biệt yếu tố chuyển động để dự đoán điểm cố định cách lấy vài vị trí đặc biệt cộng với đặc điểm bất biến khác nh−

tính chất đối xứng, song song, thẳng hàng… - Tìm tịi h−ớng giải: Từ việc dự đốn điểm cố định, tìm mối quan hệ điểm với yếu tố chuyển động, yếu tố cố định yếu tố không đổi

D−ới số toán chứng minh đ−ờng thẳng qua điểm cố định

Bài toán Cho tam giác ABC, M điểm nằm tam giác Lần l−ợt vẽ hình bình hành BMCD, AMDE Chứng minh M di động đ−ờng thẳng ME qua điểm cố định

Phân tích Tr−ớc hết thử hai vị trí M trọng tõm trực tõm tam giỏc ABC Từ ta dự đốn ME ln qua trọng tâm G tam giác ABC

Lời giải Gọi N trung điểm BC N cố định Vì BMCD hình hình hành nên N trung điểm MD

Do AN đ−ờng trung tuyến tam giác AMD

Vì AMDE hình bình hành nên ME cắt AD trung điểm P đờng nên MP đờng trung tuyến tam giác AMD

Tam giác AMD có hai đờng trung tuyến AN MP cắt G nên G trọng tâm cđa tam gi¸c AMD, suy AG= 2AN

3 Do A, N cố định nên G cố định

Vậy ME qua điểm cố định trọng tâm G tam giác ABC

Nhận xét Với tốn ta chứng minh theo h−ớng lấy G trọng tâm tam giác ABC (khi G cố định) chứng minh M, G, E thẳng hàng

Bài toán Cho hai điểm cố định B C Một điểm A thay đổi hai nửa mặt phẳng bờ BC cho A, B, C không thẳng hàng Dựng hai tam giác vuông cân ADB ACE với DA = DB; EA = EC cho điểm D C nằm khác phía đ−ờng thẳng AB, E B nằm khác phía đ−ờng thẳng AC Gọi M trung điểm DE Chứng minh đ−ờng thẳng AM qua điểm cố định

Phân tích Xét tr−ờng hợp đặc biệt tam giác ABC vng cân A, A, D, E thẳng hàng M trùng với A, tức AM qua điểm cố định đỉnh tam giác vuông cân với đáy BC

(41)

Lêi giải Dựng hình bình hành ADNE gọi H giao ®iĨm cđa DN víi CE

Theo tÝnh chất hình bình hành ta có A, M, N thẳng hµng

XÐt ΔBDN vµ ΔNEC cã BD = NE (= AD),

= o− = o− =

BDN 90 ADN 90 AEN NEC, DN = EC (= AE) nªn ΔBDN =ΔNEC (c.g.c) Suy NB = NC (1); DNB ECN.=

Lại có DH // AE

= o⇒ = o

AEC 90 NHC 90

Do DNB HNC + =HCN HNC 90 + = o ⇒BNC 90 (2) = o

Từ (1) (2) ta đ−ợc tam giác BNC vuông cân N N cố định

Vậy AM ln qua điểm N cố định

Bài tốn Cho đ−ờng tròn (O; R) dây AB cố định C điểm chuyển động cung lớn AB Gọi M trung điểm AC Chứng minh đ−ờng thẳng kẻ qua M vng góc với BC qua điểm cố định

Lời giải Vẽ đ−ờng kính BD, B cố định nờn D c nh

Giả sử đờng thẳng kẻ qua M vuông góc với BC cắt AD t¹i I Ta cã BCD 90 (gãc néi = o

tiếp chắn nửa đ−ờng trịn đ−ờng kính BD) Do MI // CD

XÐt ΔACD cã MA = MC, MI // CD

Suy I trung điểm AD cố định hay đ−ờng thẳng kẻ qua M vng góc với BC qua I cố định

Bài toán Cho tam giác ABC hai điểm M, N thứ tự chuyển động hai tia BA, CA cho BM = CN Chứng minh đ−ờng trung trực MN qua điểm cố định

Ph©n tÝch Khi M ≡ B th× N ≡ C

Do đ−ờng trung trực MN trung trực BC

Vậy điểm cố định nằm đ−ờng trung trực BC

(42)

Trung trùc cña AK cắt trung trực BC I, chứng tỏ I nằm đờng tròn ngoại tiếp tam giác ABC

Lời giải Gọi I giao điểm đờng trung trực BC MN

Xét ΔIMB vµ ΔINC cã

BM = CN (gt), IM = IN, IB = IC (v× I thuéc trung trùc cđa MN vµ BC)

Do ΔIMB =ΔINC (c.c.c) Suy MBI NCI.=

XÐt tø gi¸c ABCI có MBI NCI= nên tứ giác ABCI nội tiếp hay I thuộc đờng tròn ngoại tiếp tam giác ABC

Mà đ−ờng trung trực BC cố định nên I cố định

Vậy trung trực MN qua I cố định

NhËn xÐt Ta còng giải toán theo cách: Lấy I giao điểm đờng trung trực cạnh BC với đờng tròn ngoại tiếp tam giác ABC, chứng minh I thc ®−êng trung trùc cđa MN theo h−íng chøng minh MI = NI

Bài toán Cho đờng tròn tâm O, dây AB Điểm M di chuyển cung lớn AB Các đờng cao AE, BF tam giác ABM cắt H Đờng tròn tâm H bán kÝnh HM c¾t MA, MB theo thø tù ë C, D

a) Chứng minh đ−ờng thẳng kẻ từ M vng góc với CD ln qua điểm cố định

b) Chứng minh đ−ờng thẳng kẻ từ H vng góc với CD qua điểm cố định

Phân tích a) Ta thấy đ−ờng thẳng kẻ qua M vng góc với CD qua điểm O b) Xét hai vị trí đặc biệt M, M vị trí cho MA, MB lần l−ợt đ−ờng kính ta xác định đ−ợc vị trí điểm K

Khi ta thấy tứ giác MHOK hình bình hành đồng thời MH có độ dài khơng đổi suy K c nh

Lời giải a) Kẻ tiếp tuyến Mx với đờng tròn (O) xMO 90 = o

Ta cã xMB MAB (cïng ch¾n cung MB =

đờng tròn tâm O)

Lại có AEB AFB 90= = onên tứ giác ABEF nội tiếp đờng tròn đờng kính AB

Suy MEF MAB =

Do MEF xMB =

Mà hai góc vị trí so le nên Mx // EF

Lại có xMO 90 = o

Suy OM⊥EF (1)

Ta có H tâm đờng tròn ngoại tiếp tam giác MCD

HEMD nên E trung điểm MD Tơng tự F trung điểm MC

Suy EF đờng trung bình tam giác MCD hay EF // CD (2)

Tõ (1) vµ (2) suy OM⊥CD

Do đ−ờng thẳng qua M vng góc với CD qua điểm cố định O

b) Gọi K điểm đối xứng với O qua AB, ta có OK⊥AB trung điểm I AB

Mµ MH⊥AB suy MH // OK

Gọi N hình chiếu vuông góc O MB BN = MN

Vì OI // MH (cïng vu«ng gãc víi AB), IN // AB nên OIN HMA= (góc có cạnh tơng ứng song song), t−¬ng tù ONI HAM.=

Từ suy ΔOIN ΔHMA (g.g) Do OI = IN = 1⇒MH 2OI OK.= =

HM AM VËy tø gi¸c MHKO hình bình hành

(43)

Do O, AB cho tr−ớc, nên K điểm cố định Vậy đ−ờng thẳng kẻ từ H vng góc CD qua điểm K cố định

Nhận xét Với phần b ta chứng minh theo cách: Gọi K giao điểm đ−ờng thẳng kẻ qua H vng góc với CD đ−ờng thẳng kẻ qua O vng góc với BC, chứng minh K cố định

Bài tốn Cho góc vng xOy, điểm A cố định Ox, điểm B di chuyển Oy Đ−ờng tròn tâm I nội tiếp tam giác ABO tiếp xúc với AB, BO theo thứ tự M, N Chứng minh đ−ờng thẳng MN qua điểm cố định

Phân tích Vì ΔBMN cân B ≡ O

= o

MNB 45 nên điểm cố định phải nằm phân giác góc xOy

Khi B c¸ch O xa bán kính đờng tròn (I) gần OA

2 MN đ−ờng thẳng

song song với Ox cách Ox khoảng OA

2

Lêi gi¶i

Gi¶ sư phân giác Om góc xOy cắt MN F

Vì ΔBMN cân ONM 90= o+B Lại có AIO 90= o +B.

2 Suy ONM AIO =

Từ ta có ΔAIO ΔFNO (g.g)

⇒ OF = ON=cosION cos 45= o=

OA OI

⇒OF=OA

2 không đổi

Bài tốn Cho hình vng ABCD có tâm O Vẽ đ−ờng thẳng (d) quay quanh O cắt AD, BC theo thứ tự E, F Từ E, F lần l−ợt vẽ đ−ờng thẳng song song với BD, CA chúng cắt I Qua I vẽ đ−ờng thẳng (m) vng góc với EF Chứng minh đ−ờng thẳng (m) qua điểm cố định (d) quay quanh O

Ph©n tÝch Khi E ≡ A HI qua A vuông góc với AC

Khi E ≡ D HI qua B vng góc với BD Do tính chất đối xứng hình vẽ nên điểm cố định nằm đ−ờng trung trực AB Dự đoán điểm cố định K nằm đ−ờng trịn đ−ờng kính AB

Lêi gi¶i

DÔ thÊy I thuéc AB

(44)

Ta có IHE IAE 180+ = o nên tứ giác IHEA néi

tiÕp

Suy IHA IEA BDA 45 = = = o

T−¬ng tù IHF IBF 180+ = o nên tứ giác IHFB nội tiÕp

Suy IHB IFB ACB 45 = = = o

Vẽ đờng tròn đờng kính AB, ta cã

= + = o

BHA IHA IHB 90 nên H thuộc đờng tròn đờng kính AB

Gọi K giao điểm HI với đờng tròn đờng kính AB ta có:

= = = o

s® KA 2KHA 2IHA 90

Do K thuộc đ−ờng trịn đ−ờng kính AB số đo cung KA 90o nên K cố định hay

đ−ờng thẳng (m) qua K cố định

Bài tốn Cho đ−ờng thẳng (d) nằm ngồi đ−ờng tròn (O) I điểm di động (d) Đ−ờng trịn đ−ờng kính OI cắt (O) M, N Chứng minh đ−ờng thẳng MN qua điểm cố định

Phân tích Do M, N đối xứng với qua OI nên điểm cố định nằm đ−ờng thẳng qua O vng góc với (d)

Lời giải Kẻ OH vuông góc với (d) H, OH cắt MN P

Gọi Q giao ®iĨm cđa MN vµ OI

XÐt ΔOQP vµ ΔOHI cã gãc O chung,

= = o

OQP OHI 90

Suy ΔOQP ΔOHI (g.g)

Do OP= OQ ⇒OP.OH OQ.OI.=

OI OH

Lại có tam giác OMI vuông M có ®−êng cao MQ nªn OM2= OQ.OI

Từ ta có OP.OH OM= 2⇒OP= OM2

OH (khơng đổi O, H cố định OM bán kính (O) không đổi)

Vậy P cố định, MN qua P cố định

Bµi tËp tù lun

Bài Cho góc vng xOy A, B theo thứ tự chuyển động Ox, Oy cho OA + OB

= a (a độ dài cho tr−ớc) Gọi G trọng tâm tam giác OAB Chứng minh đ−ờng thẳng qua G vng góc với AB ln qua điểm cố định

Bài Cho đ−ờng tròn tâm (O) Từ điểm A cố định (O) kẻ tiếp tuyến AB, AC tới (O) (B, C tiếp điểm) Lấy điểm M cung nhỏ BC Gọi D, E, F theo thứ tự hình chiếu vng góc M BC, AC, AB MB cắt DF P, MC cắt DE Q Chứng minh M di chuyển cung nhỏ BC đ−ờng thẳng nối giao điểm hai đ−ờng tròn ngoại tiếp tam giác MPF MQE qua điểm cố nh

Bài Cho đoạn thẳng AB điểm M đoạn thẳng Từ M vẽ tia Mx vuông góc với AB Trên tia Mx lÊy hai ®iĨm C, D cho MC = MA, MD = MB Đờng tròn (O1) qua ba điểm A, M, C đờng tròn

(O2) qua ba điểm B, M, D cắt điểm

(45)

lời giải đợc cha?

Nguyễn Thanh Giang

Trờng THPT chuyên Hng Yên, Hng Yên

Bài toán Giả sử M điểm nằm nửa

đờng tròn (O) đờng kính AB Kẻ MH vuông

góc với AB H (H không trùng O) Đờng

tròn (O) đờng kính MH cắt MA, MB (O)

lần lợt E, F C (C kh¸c M) Chøng

minh AB, EF, CM đồng quy Một học sinh giải nh− sau:

Gọi I giao điểm EF MC, ta chứng minh A, B, I thẳng hàng hay MH vuông gãc víi HI ThËt vËy ta cã

1

HMI HMC s® HC;

1

MIH (s® HM s® HC) s® MC

2

= =

= − =

o

1

HMI MIH (s® HC s® MC) 90

⇒ + = + =

⇒MH HI (®pcm) ⊥

Các bạn nhận xét xem lời giải có chỗ ch−a ổn? Bạn sửa lại cho nhé!

(TTT2 sè 202)

kết luận nh

ch

a?

Lêi giải sai lầm chỗ

MA + MC > AC MB + MD > BD bạn không xét khả điểm M thuộc AC BD

Li gii ỳng

Gọi O giao điểm cđa AC vµ BD

Ta có MA + MC ≥ AC MB + MD ≥ BD Do MA + MB + MC + MD ≥ AC + BD Đẳng thức xảy M ≡ O

Vậy M giao điểm hai đ−ờng chéo AC BD tổng MA + MB + MC + MD đạt giá trị nhỏ

NhËn xÐt Bµi toán không khó

phỏt hin li sai, nờn có nhiều học sinh lớp tham gia giải quyết, hầu hết phát giải lại tốt, có số lời giải lại ch−a thật chặt ch

Các bạn sau có lời giải tốt nhận đợc quà kì này: Thân Lan Phơng, 9C, THCS Ngun

Cao, Q Vâ, B¾c Ninh; Ngun Thanh Tn, 8G, THCS Đặng Thai Mai, TP Vinh,

Nghệ An; Trần Minh Hoàng, 7E THCS

Nguyễn TrÃi, Nghi Xuân, Hà Tĩnh; Lu Thùy Dơng, 8A3, THCS Trần Đăng Ninh, TP Nam Định, Nam Định; Trần Trung Phúc, 8A4, THCS Ngô Gia Tự, Hồng Bàng, Hải

Phòng

Các bạn sau có lời giải đ−ợc khen: Đặng Quang Huy, 8G; Tr−ơng Quang Mạnh, 9D, THCS Đặng Thai Mai, TP Vinh; Lê Khắc H−ng, 7A, THCS Lý Nhật Quang, ụ Lng,

Nghệ An; Nguyễn Sơn Hải; Nguyễn Mạnh

Hùng, 9A3, THCS Giấy Phong Châu, Phù Ninh; Trần Thị Thanh Thảo, 8A3, THCS Lâm Thao, Lâm Thao, Phú Thọ; Lê Thị Diệu Thúy, 9A, THCS Bình Thịnh, §øc Thä, Hµ

TÜnh

(46)

những yêu cầu cần đạt lớp 7

mạch kiến thức “số đại số”

1 Sè

Sè hữu tỉ

+ Số hữu tỉ tập hợp số hữu tỉ Thứ tự tập hợp số hữu tỉ

- Nhận biết đợc số hữu tỉ lấy đợc ví dụ

về số hữu tỉ

- Nhận biết đợc tập hợp số hữu tỉ

- Biểu diễn đợc số hữu tØ trªn trơc sè

- Nhận biết đ−ợc số đối số hữu tỉ

- NhËn biết đợc thứ tự tập hợp số

hữu tỉ So sánh đợc hai số hữu tỉ

+ Các phép tính với số hữu tỉ

- Thực đợc phép tính: cộng, trừ,

nhân, chia tập hợp số hữu tỉ

- Mô tả đợc phép tính luỹ thừa với số mũ tự

nhiên số hữu tỉ mét sè tÝnh chÊt

của phép tính (tích th−ơng hai luỹ

thõa cïng c¬ sè, luỹ thừa luỹ thừa)

- Mô tả đợc thứ tự thực phép tính,

quy tắc dấu ngoặc, quy tắc chuyển vế tập hợp số hữu tỉ

- Vận dụng đợc tÝnh chÊt giao ho¸n, kÕt

hợp, phân phối phép nhân phép

céng, quy t¾c dÊu ngoc với số hữu tỉ

tính toán (tính viết tính nhẩm, tính nhanh cách hợp lí)

- Giải đ−ợc số vấn đề thực tiễn

gắn với phép tính số hữu tỉ (ví dụ: tốn liên quan đến chuyển động Vật lí, đo đạc, )

Số thực

+ Căn bậc hai số học

- Nhận biết đợc khái niệm bậc hai số

học số không âm

- Tính đ−ợc giá trị (đúng gần đúng)

bậc hai số học số nguyên dơng

bằng máy tính cầm tay

+ Số vô tỉ Số thực

- Nhận biết đợc số thập phân hữu hạn số

thập phân vô hạn tuần hoàn

- Nhận biết đợc số vô tỉ, số thực, tập hợp số thực

- Nhận biết đợc trục số thực biểu diễn

đợc số thực trục số trờng hợp

thn lỵi

- Nhận biết đ−ợc số đối ca mt s thc

- Nhận biết đợc thứ tự tập hợp số

thực

- Nhận biết đ−ợc giá trị tuyệt đối s

thực

- Thực đợc ớc lợng làm tròn số

cn c vo chớnh xác cho tr−ớc

+ TØ lƯ thøc vµ d·y tỉ số

- Nhận biết đợc tỉ lệ thức tính chất tỉ lệ thức

- Vận dụng đợc tính chất tỉ lệ thức

giải toán

- Nhận biết đợc dÃy tỉ số

- Vận dụng đợc tÝnh chÊt cña d·y tØ sè b»ng

nhau giải toán (ví dụ: chia số

thành phần tỉ lệ với số cho trớc, )

+ Giải toán đại l−ợng tỉ lệ

- Giải đ−ợc số toán đơn giản i

lợng tỉ lệ thuận (ví dụ: toán vỊ tỉng s¶n

phẩm thu đ−ợc suất lao động, )

- Giải đ−ợc số tốn đơn giản đại

l−ỵng tØ lƯ nghịch (ví dụ: toán thời gian

(47)

2 Đại số

Biu thc i s

+ Biểu thức đại số

- Nhận biết đ−ợc biểu thức đại số

- Tính đ−ợc giá trị biểu thức đại số

+ §a thøc mét biÕn

- Nhận biết đ−ợc định nghĩa đa thức

biÕn

- NhËn biết đợc cách biểu diễn đa thức

bin; xác định đ−ợc bậc đa thức

biÕn

- Tính đợc giá trị đa thức biết giá trị

của biến

- Nhận biết đợc khái niệm nghiệm đa

thức biến

- Thực đợc phép tÝnh: phÐp céng,

phÐp trõ, phÐp nh©n, phÐp chia tập

hợp đa thức biến; vận dụng đợc

nhng tớnh cht ca cỏc phộp tớnh ú tớnh toỏn

MạCH KIếN THứC HìNH HọC Và ĐO LƯờNG

1 Hình học trực quan

Các hình khối thực tiễn

+ Hình hộp chữ nhật hình lập phơng

- Mụ tả đ−ợc số yếu tố (đỉnh,

cạnh, góc, đờng chéo) hình hộp chữ

nhật hình lập phơng

- Gii quyt c số vấn đề thực tiễn

g¾n víi viƯc tÝnh thĨ tÝch, diƯn tÝch xung quanh cđa h×nh hép chữ nhật, hình lập

phơng (ví dụ: tính thể tÝch hc diƯn tÝch

xung quanh số vt quen thuc cú

dạng hình hộp chữ nhật, hình lập phơng, )

+ Lng tr đứng tam giác, lăng trụ đứng tứ giác

- Mơ tả đ−ợc hình lăng trụ đứng tam giác,

hình lăng trụ đứng tứ giác (ví dụ: hai mặt đáy song song; mặt bên hình chữ

nhật) tạo lập đ−ợc hình lăng trụ đứng tam

giác, hình lăng trụ đứng tứ giác

- Tính đợc diện tích xung quanh, thể tÝch

của hình lăng trụ đứng tam giác, hình lăng trụ đứng tứ giác

- Giải đ−ợc số vấn đề thực tiễn

gắn với việc tính thể tích, diện tích xung quanh lăng trụ đứng tam giác, hình lăng trụ đứng tứ giác (ví dụ: tính thể tích diện tích xung quanh số đồ vật quen thuộc có dạng lăng trụ đứng tam giác, lăng trụ đứng tứ giác, )

2 Hình học phẳng

Các hình hình học

+ Gúc v trớ đặc biệt Tia phân giác

gãc

- Nhận biết đ−ợc góc vị trí đặc biệt (hai

góc kề bù, hai góc đối đỉnh)

- Nhận biết đợc tia phân giác góc

- Nhận biết đợc cách vẽ tia phân giác

một góc dụng cụ học tËp

+ Hai đ−ờng thẳng song song Tiên

Euclid đờng thẳng song song

- Mô tả đợc số tính chất hai đờng

thẳng song song

- Mô tả đợc dÊu hiƯu song song cđa hai

đ−ờng thẳng thơng qua cặp góc đồng vị, cặp

gãc so le

- Nhận biết đ−ợc tiên đề Euclid đ−ờng

th¼ng song song

+ Khái niệm định lí, chứng minh định lí

Nhận biết đ−ợc định lí, chứng

minh định lí

+ Tam gi¸c Tam giác Tam giác

cân Quan hệ đờng vuông góc

ng xiờn Cỏc ng ng quy tam

gi¸c

- Giải thích đ−ợc định lí tổng góc

mét tam gi¸c b»ng 180o

- Nhận biết đ−ợc liên hệ độ dài ba

c¹nh mét tam giác

- Nhận biết đợc khái niệm hai tam giác

(48)

- Giải thích đợc trờng hợp hai tam giác, hai tam giác vuông

- Mô tả đợc tam giác cân giải thích đợc

tớnh chất tam giác cân (ví dụ: hai cạnh bên nhau; hai góc đáy nhau)

- Nhận biết đợc khái niệm: đờng vuông

góc đờng xiên; khoảng cách từ điểm

n mt ng thng Gii thớch c quan

hệ đờng vuông góc đờng xiên dựa

trờn mi quan hệ cạnh góc đối tam giác (đối diện với góc lớn cạnh

lín h¬n ngợc lại)

- Nhận biết đợc đờng trung trực

đoạn thẳng tính chất đờng

trung trực

- Nhận biết đ−ợc: đ−ờng đặc biệt

tam giác (đờng trung tuyến, đờng cao,

ng phõn giỏc, đ−ờng trung trực); đồng

quy đ−ờng đặc biệt

+ Giải tốn có nội dung hình học vận dụng giải vấn đề thực tiễn liên quan đến hình học

- Diễn đạt đ−ợc lập luận chứng minh hình

học tr−ờng hợp đơn giản (ví dụ:

lập luận chứng minh đợc đoạn thẳng

bằng nhau, góc từ điều kiện ban đầu liên quan đến tam giác, )

- Giải đ−ợc số vấn đề thực tiễn

liên quan đến ứng dụng hình học nh−:

đo, vẽ, tạo dựng hình học

Thực hành phòng máy tính với phần mềm toán học (nếu nhà trờng có điều kiện thực hiện)

- Sử dụng phần mềm để hỗ trợ việc học kiến thức hình học

- Thực hành sử dụng phần mềm để vẽ hình thiết kế đồ hoạ liên quan đến khái

niÖm: tia phân giác góc, đờng

trung trực đoạn thẳng, đờng

c bit tam giác

M¹CH KIÕN THøC “MéT Sè ỸU Tè THốNG Kê Và XáC SUấT

1 Một số yếu tố thống kê Thu thập tổ chức liệu

+ Thu thập, phân loại, biểu diễn liệu theo

các theo tiêu chí cho trớc

- Thực lí giải đợc việc thu thập,

phân loại liệu theo tiêu chí cho trớc

từ nguồn: văn bản, bảng biểu, kiến thức môn học khác thực tiễn

- Giải thích đợc tính hợp lí liƯu theo

các tiêu chí tốn học đơn giản (ví dụ: tính hợp lí, tính đại diện kết luận vấn; tính hợp lí quảng cáo; )

+ Mô tả biểu diễn liệu bảng, biểu đồ

- Đọc mô tả thành thạo liệu dạng biểu đồ thống kê: biểu đồ hình quạt trịn (pie chart); biểu đồ đoạn thẳng (line graph)

- Lựa chọn biểu diễn đợc liệu vào

bảng, biểu đồ thích hợp dạng: biểu đồ hình quạt trịn (cho sẵn) (pie chart); biểu đồ đoạn thng (line graph)

- Nhận biết đợc dạng biểu diễn khác

nhau cho tập liệu

Phân tích xử lí liệu

(49)

- Nhận đ−ợc vấn đề hoc quy lut n

giản dựa phân tích số liệu thu đợc

dng: biu hình quạt trịn (cho sẵn) (pie chart); biểu đồ đoạn thẳng (line graph)

- Giải đ−ợc vấn đề đơn giản liên

quan đến số liệu thu đ−ợc dạng: biểu

đồ hình quạt tròn (cho sẵn) (pie chart); biểu đồ đoạn thẳng (line graph)

- Nhận biết đợc mối liên hệ thống kê

với kiến thức môn học khác

trong Chơng trình lớp (ví dụ: Lịch sử

Địa lí lớp 7, Khoa học tự nhiên lớp 7, ) thực tiễn (ví dụ: môi trờng, y học, tài chính, )

2 Mét sè yÕu tè x¸c suÊt

+ Làm quen với biến cố ngẫu nhiên Làm quen với xác suất biến cố ngẫu nhiên số ví dụ đơn giản

- Làm quen với khái niệm mở đầu biến cố ngẫu nhiên xác suất biến cố ngẫu nhiên ví d n gin

- Nhận biết đợc xác suÊt cña mét biÕn cè

ngẫu nhiên số ví dụ đơn giản (ví dụ: lấy bóng túi, tung xúc xắc, )

Thùc hµnh phòng máy tính với phần mềm toán học (nếu nhà tr−êng cã ®iỊu kiƯn thùc hiƯn)

Sử dụng đ−ợc phần mềm để tổ chức liệu

vào biểu đồ hình quạt trịn (pie chart); biểu đồ đoạn thẳng (line graph)

HOạT ĐộNG THựC HàNH Và TRảI NGHIƯM

Nhµ tr−êng tỉ chøc cho häc sinh mét sè ho¹t

động sau bổ sung hoạt động khác tuỳ vào điều kiện cụ thể

Hoạt động 1: Tìm hiểu số kiến thức

tµi chÝnh:

- Thùc hµnh tÝnh toán việc tăng, giảm theo giá trị phần trăm mặt hàng kế hoạch sản xuất, kinh doanh

- Làm quen với giao dịch ngân hµng

- Lµm quen víi th vµ viƯc tÝnh thuÕ

Hoạt động 2: Thực hành ứng dụng kiến

thức toán học vào thực tiễn chủ đề liên môn, chẳng hạn:

- Vận dụng kiến thức thống kê để đọc hiểu bảng biểu Lịch sử Địa lí lớp 7, Khoa học tự nhiên lớp

- Thu thập, phân loại biểu diễn liệu

(theo tiêu chí cho tr−ớc) vào biểu đồ hình

quạt tròn (pie chart) biểu đồ đoạn thẳng (line graph) từ vài tình thực tiễn

Hoạt động 3: Tổ chức hoạt động

giờ khoá nh thực hành lớp học,

dự án học tập, trò chơi học Toán, thi Toán, chẳng hạn:

- To dng hình có liên quan đến tia phân giác góc, liên quan đến hai

đ−ờng song song, liên quan đến hình lăng

trụ đứng

- VËn dơng kiÕn thøc vỊ tam gi¸c b»ng thực tiễn, ví dụ: đo khoảng cách hai vị trí mà chúng có vật cản

đến đ−ợc hai vị trí

- Thu thập số vật thể thực tiễn có dạng hình lăng trụ đứng tính diện tích xung quanh vật thể

Hoạt động (nếu nhà tr−ờng có điều kiện

thùc hiƯn): Tỉ chøc giao l−u víi häc sinh cã

kh¶ yêu thích môn Toán

trờng trờng bạn

TTT (Tổng hợp)

(50)

đề thi

câu lạc ttt

th¸i nhËt phợng

Kì 32

CLB1 Find the natural number a such that it has exactly factors and three of the factors add up to a

CLB2 Given number x < such that

x4− 23x2+ = Find the value of 18 +

9

x

x

CLB3 Solve the following equation 6x4− 5x3− 5x −6 =

CLB4 Given that a + b + c = a2 + b2+ c2=

and x = =y z (a,b,c 0).≠

a b c

Show that (x + y + z)2= x2+ y2+ z2

CLB5 Given right triangle ABC (right angle A

and AB < AC), BM is the median line Point D

is one AC such that ∠ABD= ∠CBM Draw

DE perpendicular to BC at E Prove that

=

DA BA

DE BC

đỗ đức thành(dịch)

K× 30

(TTT2 sè 202)

CLB1.Vì x4∈Z nên 3y ∈Z Do y ∈N Nếu y = x4= 28561 = 134

⇔x = 13 hc x =−13

NÕu y > th× 3y chia hÕt cho V× 28562

chia d− nªn x4 chia d Mặt khác x4

số phơng nên x4 chia d

(m©u thuÉn) VËy (x; y) ∈ {(13; 0); (−13; 0)}

CLB2.Ta cã

− 2+ + ≥

(x y) (x xy y )

⇔(x y)(x− 3−y ) 03 ≥ ⇔x4+ y4≥x y xy +

⇔2(x4+y ) x4 ≥ 4+x y y3 + +xy

⇔2(x4+y ) (x y) (x4 ≥ + 2−xy y ) + ⇔x4+y4 ≥8(x2 −xy y ) (Do x + + y = 4)

Đẳng thức xảy chØ x = y =

CLB3.Ta cã

+ − = ⇔ + − =

x y z

1 12x 9y 6z 36 (1)

3

− + = ⇔ − + =

x y z

1 6x 3y 4z 24 (2)

3

Trõ vÕ víi vế (1) cho (2) ta đợc

+ = ⇔ + − =

6 x 12 y 10 z 12 x y z

VËy M =

CLB4 (a −b)(b − c)(c − a)S = (c − b)(a3− a2 + bc) + (a − c)(b3− b2+ ca) + (b − a)(c3− c2+ ab)

= (c − b)(a3− a2+ bc) + (a − c)(b3− b2+ ca) + [(b − c) + (c − a)](c3− c2+ ab)

= (b − c)(c3 − c2 + ab − a3 + a2 − bc) + (c − a)(c3− c2+ ab − b3+ b2− ca)

= (b − c)(c − a)(c2 + ca + a2 − b − c − a) − (b −c)(c − a)(b2+ bc + c2− b − c − a)

= − − − + −

= − − − + +

⇒ = + + ∈

2

(b c)(c a)(a b ca bc)

(a b)(b c)(c a)(a b c)

S a b c

CLB5.Ta cã BC ≤ AB + AC = − BC

Suy BC≤

2 VËy BC lín nhÊt b»ng

1

2

B trïng A hc C trïng A

L¹i cã BC2=AB2+AC2≥ 1(AB AC)+

2

⇔ ≥ − ⇔ ≥ −

⇒ ≥ −

2

2BC (1 BC) 2BC BC

BC

VËy BC nhá nhÊt b»ng −

= = 2

AB AC

2

Nhận xét Các bạn có lời giải tốt

đợc thởng kì là: Trần Trung

Phúc, 8A4, THCS Ngô Gia Tự,

Hồng Bàng, Hải Phòng; Trần Minh Hoàng,

7E, THCS Nguyễn TrÃi, Nghi Xuân, Hà Tĩnh;

Nguyễn Thanh Tuấn, 8G; Ngô Thị An Bình,

8E, THCS Đặng Thai Mai, TP Vinh, NghƯ

An

(51)

C¸c qc gia Châu

á

nguyễn thị hiền

Trờng Tiểu học Thanh Léc, Can Léc, Hµ TÜnh

rong chữ kì có chứa tên 11 quốc gia khu vực Châu Các bạn nhanh tay tìm gửi đáp án tòa soạn nhé!

Magic Words

(TTT2 sè 202)

Không giống tiếng Việt, tiếng Anh khơng có dấu Vì thế, với số từ, thay đổi thứ tự chữ cái, ta có đ−ợc từ mới, với nghĩa hồn toàn khác

1 sore → rose (hoa hång) bus sub (tàu ngầm) add dad (bố)

4 not → ton (tÊn)

5 stone → notes (ghi chó) panel → plane (m¸y bay) net → ten (m−êi)

8 low → owl (con có)

9 raw → war (chiÕn tranh)

Nhận xét Nhiều bạn có đáp án

đúng phần quà kì đ−ợc gửi tới bạn may mắn sau: Nguyễn

M¹nh Hïng, 9A3, THCS GiÊy Phong

Châu, Phù Ninh, Phú Thọ; Đinh Phơng Nga, 7A1, THCS Trng Vơng, Mê Linh,

Hà Nội; Phạm Tuấn Minh, 7B, THCS Tân

Bình, TP Tam Điệp, Ninh Bình; Hoàng

Thị Yến Linh, 9A, THCS Lê Văn Thịnh, Gia

Bình; Nguyễn Đức Anh, 7C, THCS Nguyễn Cao, Quế Võ, Bắc Ninh

Chúc bạn năm nhiều sức khỏe học thêm đợc nhiều kiÕn thøc bỉ Ých, lÝ thó

Chủ Vườn

(52)

Toán tổ hợp

trong thi chọn học sinh giỏi

và tuyển sinh vào lớp 10 chuyên

NGUYễN ĐứC TấN(TP Hồ Chí Minh) oán tổ hợp vấn đề hay khó, th−ờng

gặp kì thi chọn học sinh giỏi lớp thi vào lớp 10 chuyên toán THPT Bài viết xin đ−ợc giới thiệu bạn đọc số tốn tổ hợp qua kì thi chọn học sinh giỏi kì thi tuyển sinh vo lp 10 chuyờn toỏn

1 Bài toán giải phơng pháp phản chứng

Phng phỏp phn chứng thuộc loại chứng minh gián tiếp dùng để chứng tỏ kết luận toán cách chứng tỏ phủ định kết luận sai

Bµi toán Huyện KS có 33 công ty, huyện KV có 100 công ty Biết rằng, công ty huyện KS hợp tác với 97 công ty hun KV Chøng minh r»ng cã Ýt nhÊt c«ng ty huyện KV hợp tác với tất công ty huyện KS

(Đề thi vào lớp 10 chuyên Toán, tỉnh Khánh Hòa, năm học 2019 - 2020)

Lời giải Vì công ty huyện KS hợp tác với 97 công ty huyện KV Nếu xem hợp tác công ty huyện KS với công ty huyện KV liên kết số liên kết 33.97 = 3201 (liªn kÕt)

Giả sử tất công ty huyện KV hợp tác với nhiều 32 cơng ty huyện KS, số liên kết nhiều từ huyện KV vào huyện KS 100.32 = 3200 (liên kết)

Ta cã 3200 < 3201 (m©u thuÉn)

Vậy điều giả sử tất công ty huyện KV hợp tác với nhiều 32 công ty huyện KS sai Do có cơng ty huyện KV hợp tác với tất công ty huyện KS

Bài tốn Có 15 bạn học sinh nam 15 bạn học sinh nữ ngồi quanh bàn trịn Chứng minh ln tồn học sinh mà hai bạn ngồi cạnh bạn n

(Đề thi vào 10 chuyên Toán, THPT chuyên Hùng Vơng, tỉnh Phú Thọ, năm học 2019 - 2020)

Lời giải Giả sử tồn cách xếp 30 bạn vào bàn tròn cho khơng có bạn ngồi hai bạn nữ Gọi bạn lần l−ợt A1, A2, ., A30 Chia 30 bạn sang hai bàn tròn gồm A1, A3, , A29 A2, A4, , A30 giữ nguyên thứ tự Khi hai bàn khơng có hai bạn nữ ngồi cạnh Do số bạn nữ bàn khơng v−ợt q ⎡⎢ ⎤⎥=

⎣ ⎦

15

2 Nh tổng số bạn nữ

c hai bàn không v−ợt 14 Mâu thuẫn với giả thiết, điều giả sử sai Vậy tồn học sinh mà hai bạn ngồi cạnh bạn nữ

Bài tốn Có đoạn thẳng có độ dài lớn 10 nhỏ 210 Chứng minh đoạn thẳng ln tìm đ−ợc đoạn thẳng để ghép thành tam giác

(§Ị thi chän häc sinh giái Toán 9, tỉnh Ninh Bình, năm học 2018 - 2019)

Lời giải Gọi độ dài đoạn thẳng cho a1, a2, a3, , a8 với 10 < a1 ≤ a2 ≤ a3 ≤ ≤a8 < 210 Giả sử khơng có ba đoạn thẳng để ghép thành tam giác Ta có

(53)

thuẫn giả thiết, điều giả sử sai Do đoạn thẳng cho ln tìm đ−ợc đoạn thẳng để ghép thành tam giác

Bài tốn Cho đa giác lồi có 10 đỉnh nh− hình vẽ

(bốn đỉnh A, B, C, D đ−ợc gọi đỉnh liên tiếp đa giác) Các đỉnh đa giác đ−ợc đánh số cách tùy ý số nguyên thuộc tập hợp {1; 2; ; 9; 10} (biết đỉnh đ−ợc đánh số số, số đ−ợc đánh đỉnh khác nhau) Chứng minh ta ln tìm đ−ợc đỉnh liên tiếp đa giác đ−ợc đánh số mà tổng số lớn 21

(Đề thi chọn học sinh giỏi toán 9, tỉnh Nam Định, năm học 2018 - 2019)

Li gii Gi số khác đ−ợc đánh liên tiếp đỉnh lần l−ợt x1, x2, , x10 ∈{1; 2; 3; ; 9; 10} Giả sử khơng tìm đ−ợc đỉnh liên tiếp đa giác mà tổng số đ−ợc đánh đỉnh lớn 21 Ta có x1 + x2 + x3 + x4 ≤ 21, x2 + x3 + x4 + x5 ≤ 21, x3 + x4 + x5 + x6 ≤ 21, , x10 + x1 + x2 + x3 ≤ 21 Do (x1 + x2 + x3 + x4) + (x2 + x3 + x4 + x5) + (x3 + x4 + x5 + x6) + + (x10 + x1 + x2 + x3) ≤ 21.10 =210 Suy x1 + x2 + x3+ + x10 ≤ 52,5 mâu thuẫn với x1 + x2 + x3 + + x10 = + + + 10 = 55 Điều giả sử sai Vậy ta ln tìm đ−ợc đỉnh liên tiếp đa giác đ−ợc đánh số mà tổng số lớn 21 Bài tốn Trong buổi gặp gỡ giao l−u học sinh đến từ n quốc gia, ng−ời ta nhận thấy 10 học sinh có học sinh đến từ quốc gia

a) Gọi k số quốc gia có học sinh tham dự buổi gặp gỡ Chứng minh n<k 10+

2 b) BiÕt r»ng sè häc sinh tham dự buổi gặp gỡ 60 Chứng minh tìm đợc

nht l 15 học sinh đến từ quốc gia

(Đề thi vào 10 chuyên Toán, trờng PTNK, ĐHQG TP Hồ Chí Minh,

năm học 2019 - 2020)

Lời giải Số quốc gia có học sinh tham dự k nên số quốc gia có hai học sinh tham dự n − k Khi ta chọn k học sinh từ k quốc gia có học sinh tham dự 2(n − k) học sinh từ n − k quốc gia có học sinh tham dự

a) Gi¶ sư n≥ k 10+ ⇔ +k 2(n k) 10− ≥

2 th× tõ

ít 10 học sinh gồm k học sinh từ k quốc gia học sinh tham dự 2(n − k) học sinh từ n − k quốc gia có học sinh tham dự, ta ln có học sinh đến từ quốc gia Điều mâu thuẫn với đề bài, điều giả sử sai Do n<k 10+

2

b) Giả sử có điều trái với kết luận tốn Ta có quốc gia có học sinh tham dự quốc gia có học sinh tham dự có nhiều 14 học sinh tham dự Do

≤ + − ⇔ − ≥

+ ≥

60 1.k 14(n k) 14n 13k 60

13k 60

n

14

Ta cã

{ }

+ ≤ < + ⇒ + < +

⇒ < ⇒ < ⇒ ∈

13k 60 k 10

n 13k 60 7k 70

14

5

6k 10 k k 0;1

3 k = ta cã 60≤ <n

14 (lo¹i)

k = ta cã 73 ≤ <n 11

14 (lo¹i)

Điều giả sử sai Vậy tìm đ−ợc 15 học sinh đến từ quốc gia Bài tốn vận dụng ngun lí Dirichlet Ngun lí Dirichlet đ−ợc phát biểu đơn giản nh− sau: Nếu nhốt n thỏ vào k lồng (n, k nguyên d−ơng) tồn lồng chứa

Ýt nhÊt n

k

⎡ ⎤ ⎢ ⎥ + ⎢ ⎥

⎣ ⎦ thá KÝ hiƯu [x] lµ số

nguyên lớn không vợt x

(54)

giảng đại chúng gồm hai buổi sáng chiều buổi bạn học sinh ngồi vào cách tùy thích Chứng minh ln tồn đ−ợc hai bạn học sinh ngồi dãy với buổi sáng buổi chiều

(Đề thi vào 10 chuyên Toán, trờng thực hành ĐHSP TP Hồ Chí Minh, năm học 2019 - 2020)

Lời giải Ta có 200 = 14.14 + Theo nguyên lí Dirichlet buổi sáng tồn dãy bàn có 15 học sinh ngồi Buổi chiều, hội tr−ờng có 14 dãy, 15 = 14.1 + 15 học sinh ngồi dãy buổi sáng theo nguyên lí Dirichlet tồn học sinh ngồi dãy vào buổi chiều Vậy ln tìm đ−ợc bạn học sinh ngồi dãy với hai buổi sáng chiều

Bài tốn Trong kì thi chọn học sinh giỏi THCS cấp tỉnh, đồn học sinh huyện A có 17 học sinh dự thi Mỗi thí sinh có số báo danh số tự nhiên khoảng từ đến 907 Chứng minh chọn học sinh đồn có tổng số báo danh chia hết cho

(Đề thi chọn học sinh giỏi Toán 9, tỉnh Gia Lai,năm học 2019 - 2020)

Li giải Xét số tự nhiên, theo nguyên lí Dirichlet số có số chia cho có số d− Khi xảy hai khả sau

Nếu có số chia cho có số d−, số cịn lại chia cho có số d− lần l−ợt 0, 1, nên tổng chúng chia hết cho

Nếu có số chia cho có số d− tổng chúng chia hết cho Vậy số tự nhiên, tồn số có tổng chia hết cho

Ta chia 17 số tự nhiên khoảng từ đến 907 thành nhóm gồm số, số số Mỗi nhóm tồn số có tổng chia hết cho Gọi tổng số nhóm 3a, 3b, 3c (a, b, c N*) Còn lại 17 − 3.3 = số, số tồn số có tổng chia hết cho 3, gọi tổng số 3d (d N*) Còn lại − = số, số tồn số có tổng chia hết cho 3, gọi tổng số 3e (e N*) Trong số a, b, c, d, e tồn số có tổng

chia hết cho Khơng mât tính tổng qt giả sử a, b, c Ta có 3a + 3b + 3c = 3(a + b + c) chia hết cho Do chọn học sinh đồn có tổng số báo danh chia hết cho

Bài toán Cho 2020 kẹo 1010 hộp cho khơng có hộp chứa nhiều 1010 kẹo hộp chứa kẹo Chứng minh tìm thấy số hộp mà tổng số kẹo cỏc hp ú bng 1010 chic

(Đề thi vào 10 chuyên Toán, tỉnh Bắc Ninh,năm học 2019 - 2020)

Lêi gi¶i Gäi sè kĐo cã 1010 chiÕc hộp lần lợt a1, a2, , a1010 (ai N*, 1010)

Gi¶ sư a1 = a2 = = a1010 = th× ta cã

a1 + a2 + + a505 = 2.505 = 1010 thỏa mÃn

yêu cầu toán

Nếu a1 kh¸c a2, xÐt d·y sè a1, a2, a1 + a2, a1 + a2 + a3, , a1 + a2 + + a1009

NÕu mét c¸c sè cđa d·y chia hÕt cho 1010 th× ta cã ®iỊu cÇn chøng minh

Nếu khơng có số chia hết cho 1010, theo nguyên lí Dirichlet tồn số dãy số chia cho 1010 có số d−, chẳng hạn a1 + a2 + + ak a1 + a2 + + al (k, l N*, k < l) Khi ak

+1 + ak+2 + + al =

(a1+ a2+ + al) − (a1+ a2+ + ak) 1010 Mµ < ak+1+ ak+2+ + al< 2020

Nªn ak+1 + ak+2 + + al = 1010

Vậy tìm thấy số hộp mà tổng số kẹo hộp bng 1010 chic

Bài toán Trong hình vuông cạnh có 2019 điểm phân biệt Chứng minh tồn hình tròn bán kính

91 nằm hình vng mà khơng có điểm 2019 điểm cho

(§Ị thi chọn học sinh giỏi Toán 9, tỉnh Nghệ An, năm häc 2018 - 2019)

Lời giải Chia hình vng cho thành 2025 hình vng nhỏ có cạnh

1

(55)

nhau vµ b»ng : 2=

45 90 Gọi (C1), (C2), ., (C2025) lần l−ợt hình trịn đồng tâm với hình trịn trên, có bán kính

b»ng

91 Ta có hình trịn nằm hình vng đơi khơng có điểm chung Có 2019 điểm nên tồn hình trịn 2025 hình trịn (C1), (C2), , (C2025) không chứa điểm 2019 điểm cho

Bài toán 10 Trên mặt phẳng cho 17 điểm phân biệt khơng có ba điểm thẳng hàng Giữa hai điểm ba điểm cho ta nối đoạn thẳng đoạn thẳng ghi số nguyên d−ơng (các số ghi đoạn thẳng số nguyên d−ơng khác nhau) Chứng minh tồn tam giác có cạnh đoạn thẳng nối mà tổng số ghi cạnh tam giác chia ht cho

(Đề thi tuyển sinh vào lớp 10 chuyên Toán, trờng THPT chuyên Lê Quý Đôn, tỉnh Bà

Rịa-Vũng Tàu, năm học 2019 - 2020)

Lời giải Ta tô màu đoạn thẳng ba màu xanh, đỏ, vàng số ghi đoạn thẳng chia cho có d− 0, 1, Bài toán giải xong chứng tỏ tồn tam giác có ba cạnh màu

Gọi A 17 điểm cho Xét đoạn thẳng nối A với điểm lại gồm 16 đoạn thẳng Vì 16 = 3.5 + theo nguyên lí Dirichlet tồn đoạn thẳng màu, khơng tính tổng qt, giả sử màu xanh Gọi đầu mút đoạn thẳng B, C, D, E, F, G

Nếu tồn đoạn thẳng nối điểm B, C, D, E, F, G màu xanh, chẳng hạn BC tam giác ABC có ba cạnh màu xanh

Nếu tất đoạn thẳng nối điểm B, C, D, E, F, G có màu đỏ vàng, xét đoạn thẳng BC, BD, BE, BF, BG theo nguyên lí Dirichlet tồn đoạn thẳng màu, chẳng hạn BC, BD, BE đỏ • Nếu ba đoạn CD, DE, CE có đoạn

màu đỏ chẳng hạn CD tam giác BCD có ba cạnh tơ đỏ

• Nếu ba đoạn CD, DE, CE tơ vàng

thì tam giác CDE có ba cạnh tơ vàng Bài tốn đ−ợc giải xong

3 Bài toán giải dựa vào đại l−ợng bất biến

Khi đối t−ợng thay đổi nh−ng có tính chất khơng bị thay đổi q trình biến đổi đ−ợc gọi tính chất bất biến Phát đại l−ợng bất biến toán nhằm giúp loại bỏ kết xảy

Bài tốn 11 Bạn Bình có 19 viên bi màu xanh, 21 viên bi màu đỏ 23 viên bi màu vàng Bình thực trị chơi theo quy tắc sau: Mỗi lần Bình chọn viên bi có màu khác sơn chúng màu thứ ba (ví dụ: Nếu Bình chọn viên bi gồm viên màu xanh, viên màu đỏ Bình sơn viên bi thành màu vàng) Hỏi sau số hữu hạn lần thực trò chơi theo quy tắc trên, bạn Bình thu đ−ợc tất viên bi màu hay không? Tại sao?

(Đề thi tuyển sinh vào lớp 10 chuyên Toán-Tin, tỉnh Vĩnh Phúc, năm học 2019 - 2020)

Li giải Sau lần thực hiện, loại màu bi tăng thêm giảm Nh− hiệu số bi hai loại bi khác màu tr−ớc sau thực sơn bi có số d− chia cho Giả sử Bình thu đ−ợc tất bi màu Khi hai loại bi có hiệu số viên bi chia hết cho Mà lúc đầu hiệu hai số 19, 21, 23 không chia hết cho 3, điều mâu thuẫn Vậy bạn Bình khơng thể thu đ−ợc bi màu Bài toán 12 Trên bảng vng 2019 ì 2019, ng−ời ta điền tồn dấu + Sau thực trình đổi dấu (dấu + thành

dÊu , dấu thành dấu +) lần lợt theo

b−íc sau:

B−ớc 1: Các dịng thứ i đ−ợc đổi dấu i lần, i = 1, 2, 3, , 2019

B−ớc 2: Các ô cột thứ j đ−ợc đổi dấu 3j + lần, j = 1, 2, 3, , 2019

(56)

khi thực trình đổi du trờn

(Đề thi tuyển sinh vào lớp 10 chuyên Toán, tỉnh Quảng NgÃi, năm học 2019 - 2020)

Lời giải Theo đầu ta có dấu ghi vng dịng i cột j đ−ợc đổi dấu i + 3j + lần

Ta có (i + 3j + 1) + (i + j) = 2(i + 2j) + lẻ nên i + 3j + i + j khác tính chẵn lẻ Do vng dịng i cột j mà i + j số lẻ đ−ợc đổi dấu số chẵn lần nh− vậy,

dấu vng dấu +, cịn

ơ vng dịng i cột j mà i + j số chẵn đ−ợc đổi dấu số lẻ lần dấu ô vuông dấu −

Từ đến 2019 có 1009 số chẵn 1010 số lẻ nên số cặp (i, j) mà i + j số lẻ 1009.1010 + 1010.1009 = 2038180

Vậy số ô vuông lại mang dấu + 2038180

4 Các toán dạng khác

Bi toỏn 13 Trong mặt phẳng cho 8073 điểm mà diện tích tam giác với đỉnh điểm cho không lớn Chứng minh số điểm cho tìm đ−ợc 2019 điểm nằm cạnh tam giác có diện tích khơng lớn

(§Ị thi chän học sinh giỏi Toán 9, tỉnh Bình Định, năm học 2018 - 2019)

Lời giải Trong số tam giác có đỉnh điểm cho, ta chọn tam giác có diện tích lớn nhất, giả sử tam giác ABC SABC Qua A, B, C lần l−ợt vẽ đ−ờng thẳng song song với BC, AC, AB chúng cắt tạo thành tam giác DEF

Ta có SDEF Nếu có điểm G nằm tam giác DEF, chẳng hạn G, E nằm khác phía BC, SGBC > SABC Nh− 8073 điểm cho nằm tam giác DEF Mặt khác 8073 = 4.2018 + Do

theo ngun lí Dirichlet tìm đ−ợc 2019 điểm cho nằm cạnh tam giác ABD, ABC, EBC, ACF có diện tích nhỏ

Bài toán 14 Cho tr−ớc p số nguyên tố Trên mặt phẳng tọa độ Oxy, lấy hai điểm A(p8; 0) B(p9; 0) thuộc trục Ox Có bao nhiêu tứ giác ABCD nội tiếp cho điểm C, D thuộc trục Oy có tung độ số ngun d−ơng?

(§Ị thi tun sinh líp 10 chuyên Toán, trờng THPT Hạ Long, tỉnh Quảng Ninh,

năm học 2018 - 2019)

Lời giải Gọi C(0; c), D(0; d) (c > d > 0) V× tứ giác ABCD nội tiếp nên OD.OC = OA.OB hay cd = p8.p9 = p17

Mà c > d > p số nguyên tố, nên có cặp (c; d) thỏa mãn (p17; 1), (p16; p), (p15 ; p2); ; (p9 ; p8) Vậy có tất tứ giác thỏa mãn đề toán

Bài toán 15 Trong buổi tổ chức Lễ tun d−ơng học sinh có thành tích học tập xuất sắc huyện, ngoại trừ bạn An, hai ng−ời bắt tay An bắt tay với ng−ời quen Biết cặp (hai ng−ời) bắt tay không lần có tổng cộng 420 bắt tay Hỏi bạn An có ng−ời quen buổi lễ tuyờn dng ú?

(Đề thi tuyển sinh vào lớp 10 chuyên Toán, tỉnh Bình Thuận, năm học 2018 - 2019)

Lời giải Gọi số bạn tham dự lễ tuyên dơng x + bạn số ngời An quen y bạn (x, y N*, y x)

Số bắt tay x(x 1) : + y

Theo đề ta có x(x − 1) : + y = 420 ⇔x(x − 1) + 2y = 840

Do x(x − 1) + 2x ≥ x(x − 1) + 2y = 840 ⇔x2 + x ≥ 840 ⇔ x 29

Mà y nên 840 = x(x − 1) + 2y ≥ x(x − 1) + ⇔x(x − 1) ≤ 838 ⇔x ≤ 29

Từ suy x = 29

Khi 29(29 − 1) + 2y = 840 ⇔y = 14

(57)

sai phân pháp sách toán

chữ hán “đại thành toán học minh

Tạ Duy Phợng(Viện Toán học)

Phạm Vũ Lộc (Trung tâm Vũ trụ Việt Nam)

Đoàn Thị Lệ(Đại học Thanh Hoa, Đài Loan)

Cung Thị Kim Thành,Phan Thị ánh Tuyết

(Tiếp theo TTT2 số 200+201)

Dạng Đa phơng trình bậc

ẩn

Dạng 2.1 Sai phân

Bài 2.1.1 (trang 31a) Nay có vàng cân 12

lạng chia cho ba ng−ời không Biết ất

kém Giáp mà nhiều Bính lạng Hi

mỗi ngời nhận đợc bao nhiêu?

Giải Ta có cân = 16 lạng, cân 12 l¹ng =

60 l¹ng

Gäi x − 5, x, x + tơng ứng số vàng Bính,

ất, Giáp nhận đợc Theo ta có (x − 5) + x + (x + 5) = 60 x = 20

Vậy Bính nhận đợc 15 lạng, ất nhận đợc cân lạng, Giáp nhận đợc cân lạng

Bi 2.1.2 (trang 31b-32a) Nay có 265 thạch gạo, giao cho ba động tiên dùng Động tiên ông 20 cung, động tiên tử (trẻ em) 50 cung, động tiên nữ 110 cung Động tiên ông cung nhận nhiều động tiên tử u

Động tiên tử cung nhận đợc nhiỊu h¬n

động tiên nữ đấu Hỏi cung động dùng bao nhiêu?

Giải Ta có 265 thạch = 2650 đấu.Gọi x + 7, x, x − t−ơng ứng số đấu gạo cung động tiên ông, tiên tử tiên nữ nhận đ−ợc Theo ta có ph−ơng trình

20(x + 7) + 50x + 110 (x − 5) = 2650

⇔ x = 17

Vậy động tiên tử cung nhận thạch đấu, tổng 85 thạch; động tiên ông cung nhận thạch đấu, tổng 48 thạch; động tiên nữ cung nhận thạch đấu, tổng 132 thạch

Lêi b×nh: Bài toán phản ánh xà hội ngày xa Có thể đợc lấy từ sách toán Trung Hoa

Dạng 2.2 Sai phân lần lợt giảm

(Ngha l mi v trí giảm phần, đến cịn phần dừng)

Bµi 2.2.1 (trang 32a-32b) Nay cã vµng cân 12 lạng, muốn chia cho bốn cô gái,

dùng phép lần lợt giảm mà thành Mỗi cô

nhận bao nhiêu?

Giải Đổi cân 12 lạng = ì 16 + 12 = 92 lạng Gọi x số vàng cô út nhận đợc Ta cã x + 2x + 3x + 4x = 92 ⇔ x = 9,2 VËy c« ót nhËn lạng 2; cô ba nhận 9,2 ì

= 18,4 (1 cân lạng 4); cô hai nhận 9,2 ì

= 27,6 (1 cân 11 lạng 6); cô nhận 9,2 ì

= 36,8 (2 cân lạng 8)

Bài 2.2.2 (trang 32b) Có 1440 gấm chia cho Công, Hầu, Bá Mỗi hạng nhận bao nhiêu?

Giải Gọi x số gấm Bá nhận đợc; Hầu nhận đợc 2x; Công nhận đợc 3x Theo ta có x + 2x + 3x = 1440

⇔ 6x = 1440 ⇔ x = 240

(58)

Dạng 2.3 Sai phõn ln lt chia ụi

(Ngời đầu nhận nhiều nhÊt, ng−êi tiÕp theo

b»ng nưa)

Bµi 2.3.1 (trang 33a) Nay cã g¹o 672 th¹ch

chia cho ba ng−êi Gi¸p, Êt, BÝnh theo phÐp

chia đơi Hỏi ng−ời nhận đ−ợc bao

nhiªu?

Giải Gọi x số gạo Bính nhận đợc Khi

ất nhận đợc 2x; Giáp nhận đợc 4x Ta cã x + 2x + 4x = 672 ⇔ 7x = 672 ⇔ x = 96 VËy BÝnh nhËn đợc 96 thạch gạo, ất nhận đợc 192 thạch, Giáp nhận đợc 384 thạch gạo

Dạng 2.4 Sai phân lần lợt bốn sáu

(Bốn sáu tức ngời phần ngời

thứ hai phần, ngời thứ hai phần

ngời thứ ba phần)

Bài 2.4.1 (trang 33b) Nay phải nép tiỊn th 4227 quan m¹ch, lƯnh ba h¹ng Giáp, ất, Bính nộp theo phép bốn sáu Hỏi

ngời nộp bao nhiêu?

Giải Gọi x, y, z số tiền Giáp, ất, Bính phải nộp Theo luËt s¸u ta cã 4x = 6y, 4y = 6z Suy y= 2x,

3 = = z y x

3 Theo bµi ta cã

+ + = ⇔ =

x x x 4227,5 x 2002,5

3 (quan) Vậy Giáp phải nộp 2002 quan mạch; ất ph¶i nép 1335 quan; BÝnh ph¶i nép 890 quan

Dạng 2.5 Sai phân đắp đổi giảm dần (Nghĩa chia theo cấp số cộng với công sai d)

Bài 2.5.1 (trang 35a-35b) Chia 180 thạch

gạo cho ba ng−êi BÝnh, Êt, Gi¸p theo phÐp

đắp đổi giảm dần Biết Bính Giáp 36

th¹ch Hỏi ngời nhận đợc

gạo?

Giải Gọi x d, x, x + d số gạo Bính, ất Giáp nhận đợc Ta có (x − d) + x + (x + d) =

180 ⇔ x = 60

vµ (x + d) − (x − d) = 36 ⇔ d = 18 VËy Êt nhËn 60 th¹ch, BÝnh nhËn 42 thạch, Giáp nhận 78 thạch

Bài 2.5.2 (trang 35b-36a) Nay cã vµng 15

cân, lệnh bốn ph−ơng theo phộp p i

giảm dần mà chia, biết Đông đầu nhiều

hơn Bắc cuối cân lạng Mỗi phơng

nhận bao nhiêu?

Gi¶i Gäi x, x + d, x + 2d, x + 3d số vàng Bắc, Nam, Tây, Đông nhận đợc Vì cân lạng tơng ứng 18 lạng Theo ta có (x + 3d) x = 18 ⇔ d =

V× 15 cân 240 lạng nên ta có x + (x + d) +

(x + 2d) + (x + 3d) = 240

⇔x + (x + 6) + (x + 12) + (x + 18) = 240

⇔ x = 51

VËy B¾c nhËn 51 lạng; Nam nhận 57 lạng; Tây nhận 63 lạng; Đông nhận 69 lạng

Dạng 2.6 Sai phân thắng bán, thái bán (Thắng bán hai phần ba, thái bán ba phần t)

Bài 2.6.1 (trang 37b) Nay có Giáp, ất, Bính

3 hạng, hạng 368 ngời, đợc chia 3974

thch u go Biết hạng Bính ng−ời

nhËn phÇn, Êt nhận nh Bính thắng bán,

Giáp nhận nh ất thái bán Hỏi ngời

nhận bao nhiêu?

Giải Số gạo ngời nhóm Bính nhận đợc phần Mỗi ngời nhóm ất nhận đợc phần Mỗi ngời nhóm Giáp nhận đợc phần

Tổng số phần 368 ì (4 + + 8) = 6624 Một phần nhận 39744 : 6624 = (đấu) Giáp ng−ời nhận phần = thạch đấu; tổng 368 ì 4,8 =1766 thạch đấu; ất ng−ời thạch đấu, tổng 1324 thạch đấu; Bính ng−ời thạch đấu, tổng 883 thạch đấu

D¹ng Đa hệ phơng trình bậc

Bài 3.1 (trang 28a-28b) Nay cã b¹c 786 l¹ng tiỊn, mua gạo, mạch tổng 920 thạch Gạo thạch giá tiền phân, mạch thạch giá tiền phân Hỏi loại bao nhiêu?

Giải 786 lạng tiền = 7866 tiền Gọi x y tơng ứng số gạo số mạch Ta có

+ = =

⎧ ⎧

⎨ + = ⎨ =

⎩ ⎩

(59)

VËy g¹o 644 th¹ch, tỉng 598 l¹ng tiỊn phân; mạch 276 thạch, tổng 187 lạng tiền ph©n

Bài 3.2 (trang 29a) Nay có bạc 45 lạng tiền, mua đồng, thiếc tổng 13862 lạng Một

tiền mua đ−ợc 28 lạng đồng, tiền mua

đợc 34 lạng thiếc Hỏi loại bao nhiªu?

Giải Gọi x y t−ơng ứng số tiền mua đồng thiếc Theo ta có

+ = =

⎧ ⎧

⎨ ⎨

+ = =

⎩ ⎩

x y 458 x 285 28x 34y 13862 y 173

Vậy đồng 7980 lạng hết 28 lạng tiền; thiếc 5882 lạng hết 17 lạng tiền

Dạng Đ−a ph−ơng trình vơ định

Bµi 4.1 (trang 29a-29b) Nay cã 1337 l¹ng tiỊn, mua gạo, mạch, đậu tổng 1965 thạch Giá thạch gạo tiền phân, mạch tiền phân, đậu tiền phân Hỏi loại bao nhiêu?

Giải Gọi x, y, z tơng ứng số thạch gạo, mạch đậu mua đợc Theo ta cã

+ + = ⎧

+ + =

x y z 1965

8,5x 6, 4y 4,5z 13375

Giải hệ ph−ơng trình vô định ta đ−ợc: Gạo 822 thạch, giá 697 lạng tiền; mạch 655 thạch, giá 419 lạng tiền; đậu 488 thạch, giá 219 lạng tiền Đây đáp số [4] Còn đáp số khác

Bài 4.2 (trang 30a) Nay có bốn loại lụa (lăng, là, the, quyên) tổng giá 495 lạng

tiền, mua đợc 240 Lăng

lạng tiền, lạng tiền, the lạng tiền, quyên lạng tiền Hỏi loại bao nhiêu?

Gi¶i Gäi x, y, z, t (x, y, z, t∈ +) tơng ứng số lăng, là, the, quyên Ta cã hÖ

+ + + = ⎧

⎨ + + + =

x y z t 240

2,6x 2,3y 1,8z 1,3t 495,6

Giải hệ ph−ơng trình vơ định ta đ−ợc Lăng 72 tấm, giá 187 lạng tiền; 60 tấm, giá 138 lạng; the 60 tấm, giá 108 lạng; quyên 48 tấm, giá 62 lạng tiền Đây đáp số [4] Còn đáp số khác

Lời bình: Đây hệ hai ph−ơng trình vơ định bốn ẩn, có lẽ lần xuất

sách toán Việt, sách toán đại gặp, xem thí dụ [2], [4]

Vĩ Thanh - Các sách tốn Hán Nơm, thí dụ [4] [5] phân loại trình bày dạng toán toán khác sai phân pháp Thí dụ, tốn đ−a ph−ơng trình vơ định [4] khơng thấy có [5] (xem [2] viết này) Điều chứng tỏ tr−ởng thành toán học Việt Nam đầu kỉ XIX

Tµi liƯu dÉn

[1] Vị Hữu Bình, Phơng trình nghiệm nguyên kinh nghiệm giải, NXB Giáo dục Việt Nam, 2011

[2] Tạ Duy Phợng, Đoàn Thị Lệ, Cung Thị Kim Thành, Phan ánh Tuyết, Sai phân pháp

trong ý Trai toỏn phỏp đắc lục, Tạp chí Tốn Tuổi thơ số 183 v s 184, trang 25-26

[3] Đặng Hùng Thắng, Nguyễn Văn Ngọc, Vũ Kim Thủy, Bài giảng số häc, NXBGDVN, 2010

[4] 梁世榮 Ph¹m Gia Kû, 大成算學指明 (Đại thành toán học minh), MÃ số th viện Viện nghiên cứu Hán Nôm (một nhất): A.1555

[5] 阮有慎 Ngun H÷u ThËn,意齋算法-得錄

(60)

Questions to 10, marks each 201 − =

(A) 111 (B) 182 (C) 188 (D) 192 (E) 198

2 This rectangle is cm wide and cm tall What is its area in square centimetres?

(A) (B) 10 (C) 18 (D) 20 (E) 40

3 The table shows the number of boys and girls aged 10 or 11 in year

How many boys aged 11 are in year 5?

(A) (B) 11 (C) 21 (D) 37 (E) 46

4 The circles are in a regular rectangular pattern Some circles are hidden by the card What fraction of the circles is hidden?

(A)

3 (B)

3 (C) (D)

6 (E) 18

5 Which one of the following is the largest number?

(A) 4.05 (B) 4.45 (C) 4.5 (D) 4.045 (E) 4.54

6 What is 25% of 1? (A)

16 (B)

8 (C) (D) (E)

7 We’re driving from Elizabeth to Renmark, and as we leave we see this sign

We want to stop at a town for lunch and a break, approximately halfway to Renmark Which town is the best place to stop?

(A) Gawler (B) Nuriootpa (C) Truro (D) Blanchetown (E) Waikerie

8 This letter is first rotated by 90o clockwise and then reflected in a horizontal line It will now look like this

A U S T R A L I A N M A T H E M A T I C S

C O M P E T I T I O N A M C

Junior Division

Australian school years and Time allowed: 60 minutes

(61)

9 Edith wrote down the whole numbers from to 20 on a piece of paper How many times did she write the digit 1?

(A) (B) 10 (C) 11 (D) 12 (E) 13

10 Danny divided a whole number P by another whole number Q on his calculator and got the answer 3.125

Later, Danny forgot the two whole numbers, but he knew that both were under 30 The value of Q is

(A) (B) (C) (D) 10 (E) 25

Questions 11 to 20, marks each 11 Every row and every column of this × square must contain each of the numbers 1, and What is the value of N + M?

(A)2 (B) (C) (D) (E)

12 A piece of paper is folded in three, then a semi-circular cut and a straight cut are made, as shown in the diagram

When the paper is unfolded, what does it look like?

13 What is the value of z?

(A) 30 (B) 35 (C) 45 (D) 50 (E) 55

14 111111111= 111

(A) 11111 (B) 1001001

(C) 10001 (D) 10101 (E) 1001

15 Jill has the same number of brothers as she has sisters Her brother Jack has twice as many sisters as he has brothers How many children are in the family?

(A) (B) (C) (D) (E) 11

16 The large rectangle shown has been divided into smaller rectangles The shaded rectangle in the bottom-right corner has dimensions of cm × cm The remaining five rectangles all have the long side equal to twice the short side The smallest of these has a width of cm

What is the total area of the original large rectangle, in square centimetres?

(A) 42 (B) 44 (C) 50 (D) 56 (E) 70

17 In my dance class, 14 students are taller than Bob, and 12 are shorter than Alice Four students are both shorter than Alice and taller than Bob How many students are in my dance class?

(A) 22 (B) 24 (C) 26 (D) 28 (E) 30

(62)

At 1.05pm yesterday the washing machine displayed 2:41, namely hours and 41 minutes remaining

When did the washing machine’s countdown display happen to agree with the actual time?

(A) 2.41pm (B) 3.46pm (C) 2.23pm (D) 1.36pm (E) 1.53pm

19 A seven-digit number is in the form 20AMC19, with all digits different It is divisible by

What is the value of A + M + C? (A) (B) (C) 12 (D) 15 (E) 18

20 John, Chris, Anne, Holly, Mike and Norman are seated around a round table, each with a card with a number on it in front of them Each person can see the numbers in front of their two neighbours, and says the sum of these two numbers

John says 30, Chris says 33, Anne says 32, Holly says 38, Mike says 36 and Norman says 41 What number does Holly have in front of her?

(A) 17 (B) 18 (C) 19 (D) 23 (E) 37

Questions 21 to 25, marks each 21 On this simple system of roads, how many ways are there to get from A to B

without visiting any of the intersections more than once?

(A) (B) 10 (C) 12 (D) 14 (E) 16

22 The average time for a class of 30 mathematics students to travel to school is 21 minutes

The boys’ average is 25 minutes and the girls’ average is 19 minutes How many boys are in the class?

(A) 10 (B) 12 (C) 14 (D) 15 (E) 18

23 A cm × cm board can have cm3 cubes placed on it as shown

The board is cleared, then a number of these cubes are placed on the grid The front and right side views are shown

What is the maximum number of cubes there could be on the board?

(A) 10 (B) 11 (C) 16 (D) 17 (E) 18

(63)

When Andy crossed the finish line, Bob was 10 metres behind When Bob crossed the finish line, Chase was 10 metres behind Bob

When Andy crossed the finish line, how far behind was Chase?

(A) 21m (B) 20m (C) 19m (D) 18m (E) 17m

25 Seven squares and two equilateral triangles, all with the same side lengths, are used to form the 3-dimensional “house shape” shown

Which of the following diagrams does not show a net which can be created by cutting along some of the edges and folding the shape flat?

For questions 26 to 30, shade the answer as an integer from to 999 in the space provided on the answer sheet

Questions 26-30 are worth 6, 7, 8, and 10 marks, respectively

26 A tower is built from exactly 2019 equal rods Starting with rods as a triangular base, more rods are added to form a regular octahedron with this base as one of its faces The top face is then the base of the next octahedron The diagram shows the

construction of the first three octahedra How many octahedra are in the tower when it is finished?

27 A positive whole number is called stable if at least one of its digits has the same value as its position in the number For example, 78247 is stable because a appears in the 4th position How many stable 3-digit numbers are there?

28 When I divide an integer by 15, the remainder is an integer from to 14 When I divide an integer by 27, the remainder is an integer from to 26

For instance, if the integer is 100 then the remainders are 10 and 19, which are different

How many integers from to 1000 leave the same remainders after division by 15 and after division by 27?

29 In a list of numbers, an odd-sum triple is a group of three numbers in a row that add to an odd number For instance, if we write the numbers from to in this order,

6

then there are exactly two odd-sum triples: (4, 2, 1) and (1, 3, 5)

What is the greatest number of odd-sum triples that can be made by writing the numbers from to 1000 in some order?

30 The Leader of Zip decrees that the digit 0, since it represents nothing, will no longer be used in any counting number Only counting numbers without digits are allowed

So the counting numbers in Zip begin 1, 2, 3, 4, 5, 6, 7, 8, 9, 11, 12, ., where the tenth counting number is 11

(64)

Hỏi: Anh ơi! Năm năm Tý Anh cho câu đố Chuột khơng?

Phan T

(Lớp 6A, THCS Hoàng Xuân HÃn, Đức Thọ, Hà Tĩnh)

Đáp:

Con thích rúc, chui Là loài gặm nhấm quen mùi thức ăn

Bt nú cng rt khú khn Dựng bẫy, dùng bả để giăng góc nhà

Hái: Nh÷ng ngày Tết, anh có thích lì xì

không?

Đoàn N

(Lớp 6B, THCS Nguyễn Hiền, Nam Trực, Nam Định)

Đáp:

Trăm năm cõi ngời ta Anh thích nhận quà đầu Xuân

ít nhiều, điều khơng cần Chỉ cần chia sẻ tình thân với

Hỏi: Nếu cho sợi dây để khoanh hình đất để diện tích hình lớn nhất, em nên tạo nên hình gì?

Ngun T

(Líp 9D5, THCS Chu Văn An, Ngô Quyền, Hải Phòng)

Đáp:

ó có định lí tài tình

DiƯn tÝch lín nhÊt hình tròn thôi! Chứng minh sức So hình vuông thử, em ngồi giải

Hỏi: Trong ngày Tết muốn vui

nhng lỡ có chuyện làm em buồn sao?

Ngô N

(Líp 6/1, THCS Thµnh BÕn Tre, BÕn Tre)

Đáp:

Tui th trng tõm hn Giữ đừng có nỗi buồn em!

Vui buồn đôi lúc đan xen Hãy chia sẻ với bạn quen thân nào…

(65)

C¸c líp &

Bài 1(204+205) Chứng minh số chữ số viết hệ thập phân hai số 20152017 vµ 20152017 + 52017 lµ b»ng

Nguyễ n Ngọ c hùng (Tr−ờng THCS Hoàng Xuân Hãn, Đức Thọ, Hà Tĩnh) Bài 2(204+205) Một cửa hàng có n túi kẹo, túi có khối l−ợng Chọn tất cặp gồm túi, tính tổng khối l−ợng đ−ợc 45 kg Còn chọn tất nhóm gồm túi, tính tổng khối l−ợng đ−ợc 180 kg Hỏi cửa hàng có tất túi kẹo

trần quang vinh (Nhà Xuất Giáo dục Việt Nam) Bài 3(204+205) Cho tam giác ABC có

= o

BAC 45 Chøng minh r»ng

+ <8

AB AC BC

3

NGU N Kh¸nh nguyên (3/29E, đờng Đà Nẵng, Q Ngô Quyền, Hải Phòng) Bài 4(204+205) Cho tam giác ABC có

= o

BAC 10 , AB = cm vµ AC = cm Trên cạnh BC kéo dài, lấy điểm G cho

= o

CAG 10 Trên cạnh CB kéo dài, lấy điểm D BAD 20 = o Trên hai tia AG AD, lấy điểm E, F Tìm giá trị nhỏ tỉng BE + EF + FC

Vâ qc b¸ cẩn (Trờng Archimedes Academy, Q Cầu Giấy, Hà Nội)

1(204+205) Prove that the two numbers 20152017 and 20152017 + 52017 can be expressed in terms of the same decimal number

2(204+205) There are n Candy bags of identical mass If we choose all the sets of bags, the total mass is 45 kg If we choose all the sets of bags, the total mass is 180 kg How many Candy bags are there in the store?

3(204+205) Given triangle ABC with ∠BAC 45 Prove that = o AB AC+ < 8BC.

3

(66)

Các lớp THCS

Bài 5(204+205) Giải phơng trình

=

4

(x 2) x x x

Lại Quang THọ

(Phòng GD-ĐT Tam Dơng, Tam Dơng,

Vĩnh Phúc) Bài 6(204+205) Cho sè thùc a, b, c tháa m·n ≤a ≤ 2, ≤b ≤ 2, ≤c ≤ 2, vµ ab + bc + ca = 3abc Chøng minh r»ng

+ + ≥

+ + +

2 2 2

1 1

(a 2) (b 2) (c 2)

NGUYÔN V¡N NHO (Tr−êng THPT NguyÔn Duy Trinh, Nghi Lộc,

Nghệ An)

Bài 7(204+205) Giải hệ phơng trình

+

+ =

⎨ ⎪

= − −

4

2 4

3

x y

x xy y

2

3y 3x 3y

cao minh quang

(Tr−êng THPT chuyªn Ngun BØnh Khiªm,

VÜnh Long)

Bài 8(204+205) Cho đờng tròn (O) dây

BC cố định Điểm A di động cung lớn BC Kẻ đ−ờng cao AH đ−ờng phân giác AD tam giác ABC Gọi M giao điểm AO BC

Chøng minh r»ng +

CM CH CD

Đoàn cát nhơn (Trờng THCS Phờng Bình Định, TX An Nhơn, Bình Định)

5(204+205) Solve the following equation

− = −

4

(x 2) x x x

6(204+205) Given real numbers a, b and c such that ≤a ≤ 2, ≤b ≤ 2, ≤c ≤ 2, and ab + bc + ca = 3abc Prove that

+ + ≥

+ + +

2 2 2

1 1

(a 2) (b 2) (c 2)

7(204+205) Solve the following system of equations

⎧ +

⎪ − + =

⎨ ⎪

= − −

4

2 4

3

x y

x xy y

2

3y 3x 3y

8(204+205) Given circle (O), chord BC is fixed Point A moves along the major arc BC Draw the altitude AH and bisector AD of triangle ABC Let M be the intersection point of AO and BC

Prove that + ≥

(67)(68)

Ngày đăng: 24/02/2021, 06:23

TỪ KHÓA LIÊN QUAN

w